In der Wissenschaftsberichterstattung machen gerade wieder Geschichten über das Multiversum die Runde. Angeblich hätte man nun einen “Beweis” oder zumindest einen “Hinweis” darauf gefunden, dass unser Universum nicht allein ist sondern nur eines von vielen Universen (siehe zum Beispiel hier (WebCite) oder hier, WebCite). Wir würden in einem “Multiversum” leben und neue Beobachtungsdaten sollen das belegen.

Dabei geht es in der wissenschaftlichen Arbeit die all dem zugrunde liegt eigentlich um etwas ganz anderes. Es geht um die Messung der kosmischen Hintergrundstrahlung und den sogenannten “kalten Fleck”. Die Bezeichnungen sind eigentlich selbsterklärend: Bei der kosmischen Hintergrundstrahlung handelt es sich um Strahlung die den gesamten Hintergrund des Kosmos erfüllt. Und der kalte Fleck ist ein Fleck im Universum an dem es kalt ist. Aber auch wenn das richtig ist hat man dadurch den eigentlichen Punkt noch lange nicht verstanden, weswegen ich doch ein wenig weiter ausholen möchte.

Die Temperaturverteilung der kosmischen Hintergrundstrahlung und der "kalte Fleck" (ESA and Durham University)

Die Temperaturverteilung der kosmischen Hintergrundstrahlung und der “kalte Fleck” (ESA and Durham University)

Die kosmische Hintergrundstrahlung ist das älteste Licht das im Universum existiert. 400.000 Jahre nach dem Urknall, vor 13,8 Milliarden Jahren, war das junge Universum weit genug abgekühlt so dass sich all die freien Elektronen an die Atomkerne binden konnten. Jetzt konnte sich erstmals das Licht im Kosmos ausbreiten; zuvor sind die Lichtteilchen immer von den freien Elektronen absorbiert und an ihnen gestreut wurden. Die Strahlung breitete sich aus und ein Teil davon schwirrt heute immer noch durchs All. Da es sich um das älteste Licht handelt ist es auch unsere beste Quelle um mehr über das junge Universum zu erfahren – ich habe hier sehr ausführlich erklärt wie und was man daraus lernen kann.

Das Licht der kosmischen Hintergrundstrahlung erreicht uns von jedem Punkt des Himmels; es hat aber nicht immer exakt die gleichen Eigenschaften. Es ist generell sehr kalt und hat eine Temperatur die nur minimal über dem absoluten Nullpunkt liegt. Aber manchmal ist die kosmische Hintergrundstrahlung ein wenig wärmer und manchmal ein wenig kälter als der Durchschnitt. Das liegt daran das die Materie im frühen Universum nicht absolut gleichmäßig verteilt war. In manchen Regionen gab es ein bisschen mehr; in manchen ein bisschen weniger. Diese “Klumpen” waren wichtig denn sie übten eine stärkere Gravitationskraft auf ihre Umgebung aus und das führte dazu dass sich die ersten Strukturen und daraus später die ersten Sterne und Galaxien im Universum bildeten. Gravitation beeinflusst aber auch das Licht und führt so zu der Variation der Hintergrundstrahlung.

Es ist also nicht überraschend, dass man in der Hintergrundstrahlung kalte (und warme) Flecken findet. Das war zu erwarten und als man sie in den 1990er Jahren das erste Mal entdeckte wurde diese Tatsache auch mit dem Nobelpreis für Physik ausgezeichnet. Es war genau das was das Standardmodell der Urknall-Kosmologie vorhergesagt hat. Und bis heute stimmen die Vorhersagen der Theorie über die Variationen der Temperatur der kosmischen Hintergrundstrahlung fast exakt mit den Messungen überein.

Aber eben nur fast! 2004 entdeckte man eine Region die heute als “cold spot” (“kalter Fleck”) bekannt ist. Sie nimmt am Himmel ungefähr 5 Grad ein (ist also circa 10 mal so groß wie der Vollmond) und 0.00015 Grad kälter als ihre Umgebung. Das ist nicht viel aber mehr als zu erwarten wäre.

Computersimulation der Struktur des Kosmos

Computersimulation der Struktur des Kosmos

Die bis jetzt favorisierte Erklärung für die Existenz des kalten Flecks hatte mit sogenannten “Voids” zu tun. Das sind Regionen im Universum in denen sich nichts befindet. Beziehungsweise weniger als sonst irgendwo. Im Kosmos sind die Galaxien ja in Galaxienhaufen angeordnet und diese Haufen in langen “fadenartigen” Strukturen, den Filamenten. Zwischen den Filamenten ist nichts – das sind die Voids (siehe dazu hier und hier). Die Voids haben jetzt erst mal nichts mit der kosmischen Hintergrundstrahlung zu tun. Aber die Hintergrundstrahlung gibt es schon sehr, sehr lange. Wenn sie uns heute erreicht, dann war sie 13,8 Milliarden Jahre unterwegs und hat in der Zeit sehr viel erlebt. Sie ist durchs Universum gereist und dabei an Galaxien vorbeigeflogen – und hat vielleicht auch Voids durchquert.

Seit Albert Einstein wissen wir, dass Licht von Gravitation beeinflusst wird: Es verliert Energie, wird also rötlicher, wenn es sich durch ein starkes Gravitationsfeld bewegt und kann diese Energie wieder zurück gewinnen, wenn es das Gravitationsfeld verlässt. In etwa so wie man Energie aufwenden muss wenn man mit dem Rad einen Hügel hinauf fährt, dann aber wieder Schwung und Energie gewinnt wenn man hinunter rollt. In einem statischen Universum gleicht sich das aus, unser Universum aber expandiert. Der “Hügel” wird in dem Fall also gestreckt während das Licht unterwegs ist und der Weg nach “oben” ist kürzer als der Weg nach “unten”. Und umgekehrt geht das ganze genau so, nur das es hier jetzt ein “Loch” ist bei dem der Weg hinein kürzer als der Weg hinaus ist. Die “Hügel” sind in der Kosmologie große Ansammlungen von Materie; die “Löcher” sind die Voids. Licht das Voids durchquert hat danach also ein bisschen weniger Energie als vorher – und die Temperatur ist geringer.

Das nennt man den “Sachs-Wolfe-Effekt” und ich habe das alles vor zwei Jahren schon mal ausführlich erklärt, denn damals haben Wissenschaftler eine Arbeit veröffentlicht um mit genau diesem Effekt die Existenz des kalten Flecks zu erklären. Sie gingen davon aus, dass der kalte Fleck uns eine Region am Himmel zeigt, in der sich weniger Materie befindet als anderswo weswegen die kosmische Hintergrundstrahlung dort kälter ist.

Und jetzt sind wir endlich wieder in der Gegenwart und beim Multiversum angelangt. Denn Ruari Mackenzie von der Universität Durham und seine Kollegen haben sich die Sache jetzt nochmal genauer angesehen (“Evidence against a supervoid causing the CMB Cold Spot”). Sie haben fast 7000 Galaxien vermessen die sich in Blickrichtung des kalten Flecks befinden und das mit Galaxien in einem anderen “normalen” Teil des Himmels verglichen. Ihr Resultat: Die Verteilung ist nicht sonderlich unterschiedlich. Das Licht aus Richtung des kalten Flecks hat zwar einige Voids durchquert aber wenn man diese Effekte aufsummiert, dann reicht es nicht um die niedrige Temperatur des kalten Flecks zu erklären.

 Verteilung der Galaxien in Blickrichtung des kalten Flecks (unten) und an einem Vergleichspunkt (oben). (Bild: Durham University)


Verteilung der Galaxien in Blickrichtung des kalten Flecks (unten) und an einem Vergleichspunkt (oben). (Bild: Durham University)

Außerdem sind sie auch noch zu dem Schluss gekommen, dass der kalte Fleck höchstwahrscheinlich keine zufällige Schwankung in der Temperatur der Hintergrundstrahlung ist. Die Wahrscheinlichkeit dafür beträgt nur 2 Prozent.

Mackenzie und seine Kollegen schließen daraus dass Voids und der Sachs-Wolfe-Effekt nicht als Erklärung für den kalten Fleck dienen können. Stattdessen muss seine Existenz tatsächlich etwas mit den Bedingungen in der Frühzeit des Universums zu tun haben. Der kalte Fleck ist kein Result dessen was das Licht seit damals auf seiner Reise zu uns erlebt hat. Es ist nicht erst unterwegs kalt geworden – sondern war es von Anfang an.

Aber was genau nun der Grund dafür ist, das wissen auch Mackenzie und seine Kollegen nicht. Und die Widerlegung der bisherigen Erklärung bedeutet auch ganz explizit nicht, dass der kalte Fleck ein Beweis für das Multiversum ist! Das könnte eine Erklärung sein: Wenn es mehr als ein Universum gibt, dann könnten sich unser Kosmos und ein anderer nach ihrer gemeinsamen Entstehung noch kurzfristig gegenseitig beeinflusst haben (zum Beispiel durch ihre Gravitationskraft) und dabei den kalten Fleck erzeugt haben. Aber wie gesagt: Das ist EINE von vielen Erklärungen und noch dazu eine der etwas exotischeren. Aber natürlich auch eine die maximal spektakulär wäre und auf jeden Fall für Schlagzeilen gut ist. Besonders dann, wenn die Astronomen sie auch noch extra in ihrer Pressemitteilung herausstellen.

In der eigentlichen wissenschaftlichen Arbeit sind die Astronomen deutlich zurückhaltender:

“Man darf nicht vergessen dass der kalte Fleck auch ohne eine Supervoid immer noch durch statistische Fluktuationen und das Standardmodell der Kosmologie erklärt werden kann.”

schreiben Mackenzie und seine Kollegen dort am Ende. Allerdings! Denn die 2% die ich auch oben erwähnt habe bedeuten ja: Würde man 50 Mal ein Universum so entstehen lassen wie es das Standardmodell der Kosmologie beschreibt, dann hätte eines davon einen kalten Fleck so wie wir ihn beobachten. Eine Chance von 1:50 ist jetzt nicht sooo enorm unwahrscheinlich.

Aber natürlich ist es auch richtig, dass man die Multiversums-Hypothesen als Erklärung heranziehen kann. Ich persönlich habe im Gegensatz zu anderen Wissenschaftlern kein prinzipielles Problem mit dem Multiversum (über das ich übrigens hier sehr viel mehr geschrieben habe). Aber aus der Widerlegung einer Hypothese zur Erklärung des kalten Flecks folgt eben nicht die Gültigkeit einer anderen und noch dazu enorm exotischen Hypothese. So spannend das auch wäre…

Kommentare (129)

  1. #1 Alderamin
    22. Mai 2017

    Diesmal ist es aber nicht nur ein Hype der Tagespresse, auch in der Fachpresse wird die Erklärung durchaus ernst genommen, und der Coautor der Originalstudie wird zitiert mit:

    “The paradox [is] that the craziest-sounding of the exotic models for the explanation of the Cold Spot, the bubble universe collision model, is actually the most standard in terms of the inflation model,” says study coauthor Tom Shanks (University of Durham, UK).

  2. #2 tomtoo
    22. Mai 2017

    Ok, mal angenommen mit unserem Universum sind noch ein Haufen andere entstanden. Was würde das für unser Universum ändern ?

  3. #3 Jokep
    22. Mai 2017

    Ich versteh leider nicht ganz, warum Photonen beim Durchqueren von Voids Energie verlieren.

    Ich dachte immer, dass sie Energie verlieren, wenn sie sich aus einem Schwerefeld herausbewegen – sozusagen als “Ersatz” dafür, dass sie nicht wie normale Materie langsamer werden können.

    Das würde aber bedeuten, dass sie beim Durchqueren eines Voids länger auf die Materie außerhalb des Voids “zufallen” und dadurch Energie gewinnen müssten.

    Wo ist mein Denkfehler? Danke

  4. #4 Mr. Half Unsinn
    22. Mai 2017

    @tomtoo: In unserem Universum wäre ein Erkenntnisgewinn zu verzeichnen. 🙂

  5. #5 Florian Freistetter
    22. Mai 2017

    @Alderamin: Dass die Autoren selbst die Sache mit dem Multiversum erzählen hab ich in meinem Artikel ja auch erwähnt. Aber trotzdem gibt es in der Arbeit keine Belege die die Sache irgendwie stützen. Ok, man kann argumentieren dass die Hypothese ein wenig wahrscheinlicher wird wenn man nun weiß dass eine andere Hypothese nicht anwendbar ist. Aber eigentlich haben die Daten von Mackenzie & Co nix mit dem Multiversum zu tun und ich finde es nicht in Ordnung das sie die Medien mit ihren Aussagen ständig in diese Richtung treiben.

  6. #6 vroomfondel
    22. Mai 2017

    @Jokep mein Erklaerungsversuch: der S.W.-Effekt sagt nur etwas ueber den Unterschied der Potentialunterschiede am Uebergang Materieballung::Void und Void::Materieballung aus. Unter der Annahme, dass die Materie ueberall etwa gleich “dicht” ist, wird beim Durcheilen eines Potentialunterschiedes (egal ob +Energie oder -Energie) aufgrund der Raumausdehnung mehr Energie umgewandelt als bei der darauf folgenden mit entgegengesetztem Vorzeichen.
    Also: in die Void rein -> Energieverlust, nach Millionen Jahren aus der Void raus -> geringerer Energiegewinn.

    Die Berge und Taeler in einer flachen Raumzeit mit Raumausdehnung werden immer sanfter.

  7. #7 tomtoo
    22. Mai 2017

    @Mr. Half Unsinn

    “In unserem Universum wäre ein Erkenntnisgewinn zu verzeichnen.”

    Ok,Ok ich will ja jetzt auch garnicht darüber nachdenken ob das ein Entropie Verlust oder Gewinn für unser Universum ist.

    Aber was würde es für Gleichungen die für unser Universum gelten ändern?

  8. #8 Christopher
    22. Mai 2017

    Was mich in der Wissenschaft stört ist die Tatsache, dass alternative Theorien schnell als exotisch dargestellt werden. Ich studiere selber Physik und man traut sich kaum irgendwo mal eigene Überlegungen zu diskutieren, weil man ganz schnell in der Ecke der “exotischen” Theorien ist die sowieso belächelt werden. Vor allem wenn man wie ich unangenehme Fragen stellt. Nach meinem bisherigen Verständnis meines Physik Studiums stellt sich für mich heraus, dass wir grundlegende Dinge der Physik nicht verstanden haben. Und ja, man sollte das kosmologische Standardmodell in Frage stellen dürfen ohne das es gleich exotisch ist. Dafür braucht man auch keine Multiversum Hypothese. Bisher gibt es keinen Nachweis für dunkle Materie genauso wenig wie für dunkle Energie. Man war bisher zu keinem Zeitpunkt in der Lage auch nur ein einziges dunkle Materie Teilchen zu detektieren. Stattdessen hält man an der dunklen Materie fest wie damals an den Lichtäther. Bislang galt aber die Gleichmäßigkeit der Hintergrundstrahlung als zwingend erforderlich für das Standardmodell. Wenn sich herausstellen sollte, dass das nicht der Fall ist hat die Physik ein Problem und zwar genau dieses, dass wir etwas Grundlegendes nicht verstanden haben. Der Beweis dafür ist übrigens, dass die Quantenmechanik nicht mit der Relativitätstheorie in Einklang zu bringen ist. Mit dem fehlenden Beweis für dunkle Materie und Energie sowie der möglichen Tatsache, dass die Hintergrundstrahlung nicht überall gleichmäßig ist, ist nach meiner Ansicht das bisherige Standardmodell der Physik widerlegt.

    In meinen Augen wäre es wichtig mit neuen Theorien fairer und offener umzugehen und sich selber einzugestehen, dass die bisherigen Theorien unvollständig sind.

  9. #9 AmbiValent
    22. Mai 2017

    @tomtoo
    Kommt darauf an, ob es wirklich nur weit entfernte Gegenden gibt, in denen sich andere Universen bemerkbar machen, oder auch nähere (aber mit kleineren Effekten, so dass sie nicht bemerkt werden).

    Wahrscheinlich würde man es auch nach einem Nachweis des Multiversums so halten, dass man dieselben Gleichungen wie zuvor verwendet, nur mit Ergänzungen in den Zonen, wo sich andere Universen bemerkbar machen.

  10. #10 AmbiValent
    22. Mai 2017

    @Christopher
    Weißt du, wie viele Leute mit “eigenen Überlegungen” es gibt? So viele, dass diejenigen, die den Stand der Wissenschaft erklären wollen, unmöglich auf alle eingehen können.

    Und das sage ich als jemand, der selbst “eigene Überlegungen” anstellt. In so gut wie allen Fällen hat sich herausgestellt, dass ich falsch lag, und dort, wo ich richtig lag, war dies nicht etwa ein Fehler der Wissenschaftler, sondern einer, der durch ungenaue Darstellung der Ergebnisse der Wissenschaft entstanden ist.

  11. #11 Mr. Half Unsinn
    22. Mai 2017

    Zu wissen, dass es andere Universen gibt, würde uns sicherlich eine Richtung weisen, in die es weiter zu forschen gilt. Wie sollte man vorher wissen, ob man dann Erkenntnisse gewinnt, die uns die Gesetze unseres Universums oder darüber hinaus besser verstehen lassen?

  12. #12 tomtoo
    22. Mai 2017

    Ist mir schon klar das ich eine Dumpfbacke bin. Aber würde sich so ein Universum nicht mit überlicht von uns entfernen ? Und wie sollten wir dann daraus eine Info gewinnen können ?

  13. #13 Phero
    22. Mai 2017

    @Christopher:
    Es ist gut, dass du in deinem Studium dir noch Gedanken über eigene Erklärungen machst. Bevor man allerdings ernstzunehmende Erklärungen für Phänomene vorschlagen kann, sollten die bisherigen Erklärungen, ihre Stärken und Schwächen und die notwendige Mathematik verstanden werden. Und ich habe das Gefühl, dass du über ein paar Sachen sehr vorschnell urteilst.

    Bisher gibt es keinen Nachweis für dunkle Materie genauso wenig wie für dunkle Energie. Man war bisher zu keinem Zeitpunkt in der Lage auch nur ein einziges dunkle Materie Teilchen zu detektieren. Stattdessen hält man an der dunklen Materie fest wie damals an den Lichtäther.

    Es konnten zwar noch keine dunklen Materieteilchen detektiert werden, aber das heißt nicht, dass es keinen Nachweis gibt. Indirekte Hinweise können ebenso gut wie direkte sein und die indirekten Hinweise auf Dunkle Materie sind gewaltig. Keine andere bisher vorgeschlagene Hypothese kann diese Phänomene alleine erklären. Übrigens ist der Lichtäther ja ein schlechtes Beispiel: Nach dem Michelson-Morley Experiment und der speziellen Relativitätstheorie, die den experimentellen Gegenbeweis und den dazugehörigen theoretischen Unterbau lieferten, wunderte die Ätherhypothese schnell aufgegeben.

    Bislang galt aber die Gleichmäßigkeit der Hintergrundstrahlung als zwingend erforderlich für das Standardmodell. Wenn sich herausstellen sollte, dass das nicht der Fall ist hat die Physik ein Problem und zwar genau dieses, dass wir etwas Grundlegendes nicht verstanden haben. Der Beweis dafür ist übrigens, dass die Quantenmechanik nicht mit der Relativitätstheorie in Einklang zu bringen ist.

    Gleichmäßig innerhalb gewisser Grenzen – und auch nur für das Standardmodell der Kosmologie (es gibt auch eins der Teilchenphysik, ich habe das Gefühl, dass du diese durcheinander bringst). Die Kosmologie sagt im Gegenteil ja sogar Fluktuationen vorher und man kann sogar ihre Stärke berechnen – und bis auf den Kalten Fleck sind die Messungen im Einklang mit der Vorhersage.
    Desweiteren können nur QM und allgemeine Relativitätstheorie bisher nicht in Einklang gebracht werden. Das heißt nicht, dass es in Zukunft nicht gelingt. Und wenn wirklich nicht: So what? QM und Relativitätstheorie machen alleine und zusammen (QFT) zahllose überprüfbare Vorhersage, die bisher die Theorien bestätigen. Das heißt nicht, dass sie auf jeden Fall die letztliche, allgemeine Theorien sind (das behauptet auch niemand) – aber sie deswegen komplett über Bord zu schmeißen ist genauso unsinnig und sei es aus praktischen Gründen. Mit Newton wird auch noch gerechnet und das sehr gut.

    Mit dem fehlenden Beweis für dunkle Materie und Energie sowie der möglichen Tatsache, dass die Hintergrundstrahlung nicht überall gleichmäßig ist, ist nach meiner Ansicht das bisherige Standardmodell der Physik widerlegt.

    Wie oben aufgeführt: Nein, ist es nicht. Der bisher fehlende Nachweis für DM steht auch im Einklang mit den Vorhersagen (wenig wechselwirkend und so…)

    In meinen Augen wäre es wichtig mit neuen Theorien fairer und offener umzugehen und sich selber einzugestehen, dass die bisherigen Theorien unvollständig sind.

    Letzteres wird dir jeder Physiker bestätigen. Aber unvollständig heißt nicht falsch! Und mit neuen Ideen wird offen umgegangen, grab dich mal durch Paper. Und fair bedeutet, dass auch mit alten Theorien fair umgegangen wird und das bedeutet: Neue Ideen müssen die bisherigen Phänomene besser ode zumindest genauso gut erklären. Das ist die Anforderung.
    Vielleicht ist die Physik auf dem falschen Dampfer, vielleicht sind wir in einer Sackgasse und brauchen neue Ideen. Letzteres sowieso immer. Aber da finden wir nicht heraus, indem wir ohne zwingende Gründe bewährtes über Bord werfen. Schon häufig stellte sich heraus, dass eine Erweiterung einer bestehenden Theorie die bessere Lösung war als ein komplettes Neuerfinden des Rades. Beides wird gebraucht – aber ein neues Rad muss dann wenigstens so gut Rollen wie das alte. Oder zumindest die Hoffnung erwecken, dass es das mit ein paar Verbesserungen könnte.

  14. #14 tomtoo
    22. Mai 2017

    @Phero

    Danke ! Das war ein schöner Einblick.

  15. #15 Christopher
    22. Mai 2017

    @Phero

    Danke schon mal für deine Antwort. Im Großen und Ganzen stimme ich dir auch zu. Ich bin natürlich auch nicht in der Lage eine bessere Theorie vorzustellen. Meine Intention war eher zu erkennen, wenn man sich auf dem Holzweg befindet. In diesem Zusammenhang stört es mich sehr, dass die QM für so viel Verwirrung sorgt und nicht in Einklang mit der Relativitätstheorie steht. Wenn dann in der Kombination noch Zweifel im Zusammenhang mit der kosmischen Hintergrundstrahlung entstehen und seit Jahren der direkte Beweis von dunkler Materie nicht gelingt kommen mir große Zweifel auf.. Allein der Gedanke, dass sich Messergebnisse bei Beobachtung verändern ist völlig paradox und keiner kann das vernünftig erklären… Übrigens hatte kein geringerer als Einstein selber große Zweifel an der QM.

    Hinsichtlich der dunklen Materie stimme ich dir natürlich zu. Die indirekten Beweise sind ohne jeden Zweifel eindeutig, dass es große Massen sein müssen die z.B. den Gravitationslinseneffekt erzeugen. Deswegen wäre innerhalb unseres Universums die dunkle Materie natürlich die vermeintlich leichteste Lösung. Anhand der Bewegungen von Galaxien usw. kann man ja relativ einfach ausrechnen welche Massen man für bestimmte Bewegungen von großen Objekten wie Galaxien braucht. Soweit so gut. Dass die Masse vorhanden ist die die Galaxien bewegt legt die dunkle Materie nahe. Aber ich stell eine Hypothese auf die hier keiner widerlegen kann, da bislang kein einziges DM Teilchen detektiert werden konnte: Das Universum ist sehr flach. Gesetzt der Fall beim Urknall sind mehrere Universen entstanden die Flach übereinander liegen ist es möglich, dass die übereinanderliegenden Universen sich durch Gravitation bemerkbar machen. Dementsprechend könnte z.B. die Masse in einem flach darüber oder darunter liegenden Universum gravitative Auswirkungen auf unser Universum haben. Solange man dunkle Materie nicht _direkt_ nachweisen kann ist die in Kurzform beschriebene Theorie genauso möglich. Mir gefällt es nicht, dass eine solche Theorie schnell in der Schublade “exotisch” landet. Abhängig vom Blickwinkel des Betrachters könnte die DM genauso exotisch sein. Der Mensch macht seit Jahrtausenden den Fehler anzunehmen, dass unsere Sonne, unsere Galaxie und unser Universum das einzige sein muss. Aufgrund unserer wissenschaftlichen Erkenntnisse der letzten hundert Jahre gehe ich mit einer Wahrscheinlichkeit von 99% davon aus, dass es Milliarden andere Universen gibt die beim Urknall entstanden sind. Das wäre einfach nur eine logische Konsequenz aus den bisherigen Erkenntnissen in der Physik. Es ist ja auch nicht nur die Milchstraße entstanden sondern Milliarden andere Galaxien. Die Annahmen, dass es nur eine Sonne, eine Galaxie oder gar nur ein Universum gibt widersprecht doch allen Erkenntnissen der letzten hundert Jahre bei denen man rausfand, dass unsere Galaxie, Sonnensystem, Stern usw. der kosmologische „Normalfall“ ist und milliardenfach vorhanden. Genauso ist es vermutlich der kosmologische Normalfall, dass es neben unserem Universum Milliarden andere gibt. Eigentlich würde ich mir für Theorien in diese Richtung nur mehr Offenheit wünschen ohne das man sofort in der Schublade exotisch landet.

  16. #16 AmbiValent
    22. Mai 2017

    @Christopher
    Du machst den Kardinalfehler, anzunehmen, eine Hypothese sei überlegen, weil man sie nicht testen und widerlegen kann. Es ist fast genau umgekehrt: eine Hypothese, die man nicht testen kann, ist so gut wie wertlos.

    Allerdings ergäbe sich aus deiner Annahme, dass die Gravitationskraft nicht von Dunkler Materie in diesem Universum, sondern von nornaler Materie in einem Universum nebenan erzeugt wird, ein etwas unterschiedliches Ergebnis.

    Dunkle Materie wäre nämlich allgemein dünn verteilt, herkömmliche Materie dagegen “verklumpt” zu Sternen und anderen Objekten, die sich dort, wo sie sind, stärker bemerkbar machen als amderswo. Selbst wenn es nicht nur ein oder zwei Universen sind, die auf unseres einwirken, sondern viele, müsste man die “Schatten” von Sternen in anderen Universen immer noch messen können. Bis jetzt ist das aber nicht geschehen.

  17. #17 Frantischek
    22. Mai 2017

    Wenn ich das richtig verstehe gibt Christophers These eins zu eins das Randall Sundrum Modell wieder.
    https://de.wikipedia.org/wiki/Randall-Sundrum-Modell
    https://de.wikipedia.org/wiki/Lisa_Randall#Lisa_Randalls_Gedankengeb.C3.A4ude

    Soweit ich weiß ist das aber bereits widerlegt oder extremst unwahrscheinlich geworden weil bis jetzt keine Kaluza Klein Teilchen gefunden wurden, und das hätten sie bei den derzeitigen Energien am LHC schon werden sollen.

  18. #18 Phero
    22. Mai 2017

    Hallo Christopher,

    alleine das Gefühl, sich auf dem Holzweg zu befinden, ist aber noch kein Argument für irgendetwas. Das Universum verhält sich nicht so wie es sich der gesunde Menschenverstand vorstellt. Zu deinen Gründen (QMRelativitätstheorie, kosmische Hintergrundstrahlung und DM) habe ich ja bereits etwas geschrieben.

    Allein der Gedanke, dass sich Messergebnisse bei Beobachtung verändern ist völlig paradox und keiner kann das vernünftig erklären… Übrigens hatte kein geringerer als Einstein selber große Zweifel an der QM

    Messergebnisse verändern sich nicht bei Beobachtung, sondern die Messung beeinflusst die Messung. Zumindest in der üblichen, Kopenhagener Interpretation der QM – und das ist auch die von Einstein kritisierte. Er hat nicht die komplette QM abgelehnt, die zu seiner Zeit schon viele richtige Vorhersagen gemacht hat. Im Übrigens ein Autoritätsargument 😉

    So, jetzt zu deiner Hypothese. Schon die Aufstellung einer Hypothese, “die hier keiner widerlegen kann” ist nicht wissenschaftlich. Wissenschaftliche Theorien müssen prinzipiell widerlegbar sein. Unwiderlegbare Hypothesen aufstellen, die alles erklären, kann ich auch. Machen die Religionen schon seit langem.

    Allerdings denke ich nicht, dass deine Hypothese nicht unwiderlegbar ist. Erst einmal wäre da die Frage: Wieso konzentriert sich denn dann die Materie der Gegengalaxien zufällig immer da, wo sich bei uns die Materie konzentriert? Außerdem erklärt dass nicht die Messungen der Hintergrundstrahlung, die ebenfalls DM hervorsagen.

  19. #19 Christopher
    22. Mai 2017

    @AmbiValent

    Nein, das war natürlich keine Hypothese von mir. Es war nur die Beschreibung einer theoretischen nicht völlig unrealistischen Möglichkeit einer Gegenhypothese zur DM. Ich würde mich selber übrigens freuen, wenn DM Teilchen endlich detektiert und somit nachgewiesen werden können. Es würde mehr Gewissheit bringen und die künftige Forschung leichter machen. Jeder Wissenschaftler ist sich (hoffentlich) bewusst, dass das Modell mit der DM zwar sehr wahrscheinlich, aber dennoch bisher nicht 100% nachgewiesen ist.

    @Frantischek

    Der Nachweis von Hypothesen in diese Richtung ist schwierig bis unmöglich. Solche Theorien wird man wahrscheinlich ohnehin nur indirekt nachweisen können. In Richtung Randall Sundrum Modell gibt es ja mehrere modifizierte Theorien. Insofern ist es natürlich nicht so abwegig, wenn andere Wissenschaftlicher sich auch Gedanken in die Richtung gemacht haben.

    Hinsichtlich des Nachweises sei allerdings auch gesagt: Die Test- und Messmethoden und die entsprechende Technik dafür ist noch lange nicht perfekt. Um solch komplexe Theorien zu bestätigen oder widerlegen bedarf es nach meiner Ansicht besserer Messmethoden. Nur weil z.B. ein bestimmtes Teilchen nicht oder noch nicht am LHC detected wurde heißt es ja nicht automatisch, dass es nicht existiert. Das ist ein ähnliches Problem wie bei der DM. Bisher hat man kein einziges DM Teilchen detektieren können. Und solange das nicht der Fall ist sollte man auch offen für andere Theorien sein. Ich bin vielleicht auch etwas ungeduldig. Aber weil man schon so lange versucht DM Teilchen zu detektieren und keine findet bin ich halt etwas skeptisch und die Skepsis steigt, wenn dann noch Ungereimtheiten bei der kosmischen Hintergrundstrahlung auftreten die eigentlich in dieser Form nicht existieren dürften. Wenn man dann noch die Probleme der QM mit einbezieht, dass sich sogar Ergebnisse durch die bloße Anwesenheit eines Beobachters verändern und die Relativitätstheorie damit nicht in Einklang zu bringen ist wird’s schwierig. Deswegen hatte ich in meinem ersten Kommentar drauf hingewiesen, dass ich glaube, dass wir etwas Grundlegendes nicht verstanden haben. Wir bräuchten also mal wieder einen neuen Einstein der die Physik voranbringt. Ich denke die meisten sind sich einig, dass sich die Physik momentan ein Stück weit in der Sackgasse befindet. Das würde sich allerdings ändern, wenn man z.B. ein DM Teilchen endlich detektieren würde.

  20. #20 Christopher
    22. Mai 2017

    @Phero

    Mit “Beobachtung” meinte ich natürlich, dass die Messung das Ergebnis verändert. Da habe ich mich etwas blöd ausgedrückt, aber ich denke trotzdem, dass du weißt was ich damit sagen wollte. Mir ist natürlich bewusst, dass man die Teilchen erst mit einem Messgerät detektieren muss und dann ist das Resultat ein anderes als wenn man kein Messgerät dazwischen hätte. Das ändert ja nichts an der Tatsache, dass eine solche Theorie in meinen Augen völlig unlogisch ist. In jedem Fall sind Teile der QM nicht schlüssig. Und die Kopenhagener Deutung ist wie der Name schon sagt auch nur eine Beschreibung für eine mögliche Deutung dieses seltsamen Phänomens.

    Wie bereits erwähnt wollte ich hier keine Hypothese aufstellen. Es sollte lediglich exemplarisch als Hypothese auf Basis anderer wissenschaftlicher Theorien und Hypothesen darstellen, dass es Alternativen gibt die bereits ebenfalls Wissenschaftlich erforscht werden, allerdings leider mit weniger Zuspruch.

  21. #21 derdeet
    22. Mai 2017

    Ein Diskutant hier klingt doch sehr nach alternativer Wissenschaft – um es vorsichtig zu formulieren . . . 😉

  22. #22 Christopher
    22. Mai 2017

    @derdeet

    Warum so ein abwertender Kommentar ohne ein Argument vorzutragen? Genau solche Beiträge braucht man nicht. Ich stelle hier keine alternative Wissenschaft vor. Nichts liegt mir ferner. Genau da liegt doch das Problem. Die Punkte die ich beschrieben habe sind Gegenstand aktueller Forschung wofür es von renommierten Wissenschaftlern entsprechende Hypothesen gibt. Daran ist überhaupt nichts alternativ.

  23. #23 HF(de)
    22. Mai 2017

    @Christopher: das liegt z.B. an Äußerungen wie dieser:

    Und solange das nicht der Fall ist sollte man auch offen für andere Theorien sein.

    Das schreiben “alternative Wissenschaftler” ganz gerne.

  24. #24 Frieda Blum
    22. Mai 2017

    Sofern es andere Universen gibt, befinden diese sich zwangsläufig in einer anderen Dimension oder befinden diese sich im selben Raum, nur noch etwas weiter entfernt?

  25. #25 Phero
    22. Mai 2017

    @Christopher

    Ich glaube, mit Aussagen wie

    Jeder Wissenschaftler ist sich (hoffentlich) bewusst, dass das Modell mit der DM zwar sehr wahrscheinlich, aber dennoch bisher nicht 100% nachgewiesen ist.

    rennst du hier offene Türen ein. Die meisten Diskutanten hier werden sich dessen bewusst sein. Wobei die DM inzwischen schon als ziemlich sicher eingeschätzt wird, nichtsdestoweniger wäre ein Teilchennachweis natürlich interessant (alleine, um mehr über sie herauszufinden). Aber die Belege sind bereits jetzt sehr überzeugend. Natürlich kann (und wird) es noch mehr geben, aber dennoch wird es DM geben. Es sei denn, eine andere Hypothese erklärt alle Beobachtungen ähnlich gut. Letztendlich recht unwahrscheinlich. Aber man kann sowohl bisherige Hypothesen weiter verfolgen als auch neue aufstellen.

    Mein Problem ist übrigens diese Aussage:

    Aber weil man schon so lange versucht DM Teilchen zu detektieren und keine findet bin ich halt etwas skeptisch und die Skepsis steigt, wenn dann noch Ungereimtheiten bei der kosmischen Hintergrundstrahlung auftreten die eigentlich in dieser Form nicht existieren dürften. Wenn man dann noch die Probleme der QM mit einbezieht, dass sich sogar Ergebnisse durch die bloße Anwesenheit eines Beobachters verändern und die Relativitätstheorie damit nicht in Einklang zu bringen ist wird’s schwierig.

    Du wirfst hier eine Menge an Sachen ineinander, die eigentlich nicht direkt zusammenhängen – oder zumindest nur recht entfernt. Und mir ist immernoch nicht klar, welche Ungereimtheiten bei der kosmischen Hintergrundstrahlung nicht existieren können.

    Ich finde es auch interessant, dass du die Probleme der Physik anscheinend vor allem daran festmachst, ob DM detektiert wird oder nicht.

    Mit “Beobachtung” meinte ich natürlich, dass die Messung das Ergebnis verändert. Da habe ich mich etwas blöd ausgedrückt, aber ich denke trotzdem, dass du weißt was ich damit sagen wollte. Mir ist natürlich bewusst, dass man die Teilchen erst mit einem Messgerät detektieren muss und dann ist das Resultat ein anderes als wenn man kein Messgerät dazwischen hätte. Das ändert ja nichts an der Tatsache, dass eine solche Theorie in meinen Augen völlig unlogisch ist. In jedem Fall sind Teile der QM nicht schlüssig. Und die Kopenhagener Deutung ist wie der Name schon sagt auch nur eine Beschreibung für eine mögliche Deutung dieses seltsamen Phänomens.

    Erst einmal lesen hier ja noch andere mit, schon alleine daher wollte ich das richtigstellen. Der zweite Satz trifft es aber wieder nicht richtig: Ohne Messung gibt es überhaupt kein Resultat, also auch kein “anderes”. Und “unlogisch” ohne weitere Begründung ist kein gutes Argument. Weiterhin wiederhole ich mich auch noch einmal: Was du hier angreifst, ist die Kopenhagener Interpretation der QM. Andere Interpretationen lassen dieses Messproblem garnicht erst aufkommen (Bohmsche Mechanik, Viele-Welten-Theorie). Es ist durchaus interessant, sich damit zu beschäftigen, aber ich habe den Eindruck, dass dir noch ein wenig Wissen fehlt. Darf ich mal ganz indiskret fragen, in welchem Semester du bist? Was nicht unbedingt etwas mit fehlendem Wissen zu tun hat.

  26. #26 PDP10
    22. Mai 2017

    @Frieda Blum:

    Sofern es andere Universen gibt, befinden diese sich zwangsläufig in einer anderen Dimension oder befinden diese sich im selben Raum, nur noch etwas weiter entfernt?

    So etwas wie “andere Dimensionen” gibt es nicht.

    Florian hat da mal einen schönen Artikel drüber geschrieben:

    Was sind Dimensionen

    Und nein, diese Universen befänden sich auch nicht im “selben” Raum.

    Kategorien wie “sich irgendwo befinden” oder Begriffe wie “Raum” sind aus unserer Alltagswelt und machen in diesem Zusammenhang keinen Sinn.

    Und obwohl ich mal Physik studiert habe, ist das schon alles, was ich dazu sagen kann.
    Dieses ganze String-Theorie Gesumms und die höheren Weihen der Kosmologie wie auch der Elementarteilchen-Physik haben mir nämlich damals meine intellektuellen Grenzen gezeigt.

    Alles was ich darüber sagen könnte wäre:

    Diese ganzen Universen wären wohl so etwas wie Untermannigfaltigkeiten in einer 11-Dimensionalen Algebra (oder auch einer 10-Dimensionalen oder 12-Dimensionalen oder whatever). Damit hörts dann aber auch schon auf …

    Und das Verständnis befördert das auch nicht … :-).

  27. #27 Alderamin
    22. Mai 2017

    @Florian

    Die Suche nach solchen “Imprints” in der Hintergrundstrahlung ist schon länger im Gange. Die Sache ist halt die, dass ein solches Muster für alle Zeiten die einzige relativ direkte Nachweismöglichkeit bleiben wird, zu belegen, dass es mehr als ein Universum geben könnte (lediglich die charakteristischen Polarisationsmuster der Inflation können einen indirekten zusätzlichen Nachweis für “eternal inflation” liefern). Wenn nun dieser Fund nichts wert ist, dann dürften Multiversen für alle Zeiten unbelegbar und damit unwissenschaftlich bleiben.

    Die Kosmologen sollten sich vielmehr einmal diesen kalten Fleck und sein Temperatur- oder Polarisationsprofil genau anschauen und prüfen, ob sie zu Vorhersagen über ein zu erwartendes Kollisionsmuster passen könnten. Warum soll der Autor nicht darauf hinweisen, dass er die Stelle für vielversprechend hält, weil ihm andere Erklärungen weniger plausibel erscheinen?

  28. #28 jester
    22. Mai 2017

    “Es ist also nicht überraschend, dass man in der Hintergrundstrahlung kalte (und warme) Flecken findet. Das war zu erwarten und als man sie in den 1990er Jahren das erste Mal entdeckte wurde diese Tatsache auch mit dem Nobelpreis für Physik ausgezeichnet.”

    Versteh ich nicht und was hat die “tiefinelastische Streuung” mit kosmischer Hintergrundstrahlung zutun ?

  29. #29 AmbiValent
    22. Mai 2017

    @PDP10, Frieda Blum
    Es gibt verschiedene Arten, wie sich zusätzliche Dimensionen zeigen können. Soweit ich weiß, ist zur Zeit die Hypothese der “aufgerollten Dimensionen” die führende. Demnach gäbe es die Raum- und Zeitdimensionen, die wir makroskopisch erfassen können, aber zusätzlich gäbe es noch Dimensionen von beschränkter Ausdehnung, die sich aber eher nur im extremen Mikrokosmos zeigen. Das würde nicht verschiedene Universen bedeuten, sondern eher ein 4dimensionales Universum mit einer 10dimensionalen Feinstruktur.

    Aber ich denke, um dieses Modell ging es bei diesem Thema überhaupt nicht. Im oben erwähnten Randall Sundrum Modell dagegen wäre unser bekanntes Universum eine “Brane” (sozusagen Membran, im übertragenen Sinn eine Fläche), und es gäbe einen “Bulk” (im übertragenen Sinn einen Raum) – und nur Gravitation könnte den Bulk durchqueren, um eine andere Brane zu erreichen.

    Diese zusätzliche Ausdehnung des Bulks “senkrecht” zur Brane wäre dann eher das, worauf Frieda hinauswollte, als sie von Dimension sprach. Aber bewiesen ist das Randall Sundrum Modell auch nicht.

    Frieda, korrigiere mich bitte, wenn du etwas anderes meinst, aber ich habe deinen Beitrag so verstanden, dass du fragst, ob das Universum überall ähnlich aussieht bzw zu einem ähnlichen Zeitpunkt mit der Inflation aufgehört und zu dem heute beobachteten Verhalten übergegangen ist oder ob irgendwo die Inflation weitergegangen oder das Universum dort zu einem anderen Verhalten übergegangen ist als dem, was wir bei uns beobachten.

  30. #30 jester
    23. Mai 2017

    “Demnach gäbe es die Raum- und Zeitdimensionen, die wir makroskopisch erfassen können”
    Raum ja, Zeit – da scheiden sich auch die Geister.

  31. #31 Florian Freistetter
    23. Mai 2017

    @alderamin: “Wenn nun dieser Fund nichts wert ist,”

    Wertlos ist er auf keinen Fall! Aber halt zum aktuellen Zeitpunkt nicht geeignet um die Multiversumshypothese zu belegen. Da braucht es mehr Daten; müssen mehr Alternative ausgeschlossen werden; genauere Vorhersagen gemacht werden usw.

  32. #32 Artur57
    23. Mai 2017

    Anderer Ansatz: die Hintergrundstrahlung entstand ja, als sich das ursprüngliche Plasma zu den ersten H2-Atomen zusammen tat. Dabei entsteht Strahlung mit einer Temperatur von 3000K, die sich auf 3K “abgekühlt” hat, wie es vereinfachend heißt. Aber Strahlung kühlt nicht ab, vielmehr ist es so, dass jedes einzelne Photon die Raumdehnung infolge Expansion des Universums mitbekommen hat und das hat seine Frequenz entsprechend vermindert. Um dieses Thema wird immer ein kleiner Bogen gemacht, aber ich denke schon dass ich da mainstream-konform platziert bin.

    Ja nun, die Raumdehnung wird durch die im Universum befindlichen Massen bestimmt, ich halte da die Hubble-Konstante für einen sehr wichtigen Faktor. Wenn diese lokal abweicht, dann wird wohl auch die Raumdehnung etwas verschieden sein.

    Also ich muss ja schon sagen, die Raumdehnung ist ja der Haupteffekt und den gilt es zu betrachten. Erst wenn diese Methode versagt, kann man so etwas wie Multiversen in Betracht ziehen.

  33. #33 Captain E.
    23. Mai 2017

    @Christopher:

    Mal ganz ehrlich, wenn du mit dieser Art der Argumentation versuchst, deinen Professoren und ihren wissenschaftlichen Assistenten und Mitarbeitern zu verklickern, dass sie allesamt auf dem Holzweg sind, brauchst du dich wirklich nicht zu wundern, wenn man dich in die Spinner-Ecke stellt.

    Bedenke bei deiner Kritik an der modernen Physik aber auch dieses: Die Dunkle Materie wurde nicht nachgewiesen, auch wenn man viele Hinweise darauf gefunden hat, dass es sie geben muss. Das Problem ist aber halt, dass sie als “dunkel” postuliert ist. Sie wechselwirkt über die Gravitation und vielleicht (bzw. hoffentlich!) über die schwache Kernkraft. Sie wechselwirkt definitiv nicht über Elektromagnetismus und meines Wissens nach auch nicht die starke Kernkraft. Wie will man so etwas auftreiben? Der Punkt ist natürlich, wie ich an einer solchen Stelle immer wieder gerne anfüge, dass man schon früher Teilchen postuliert hatte, die über die Gravitation und die schwache Kernkraft wechselwirken sollten. Auch damals war direkt klar, dass der Nachweis schwierig werden würde, aber man hat es irgendwann geschafft und kann nun Neutrinos nachweisen. Diese “Geisterteilchen” erfüllen also recht gut die Anforderungen an die Dunkle Materie, nur leider sind sie zu schnell und bei weitem nicht zahlreich genug. (Frage: Ein Vorgang, der Neutrinos zerstörungsfrei abbremsen könnte, ist vermutlich auch nicht bekannt?) Meines Wissens gibt es aber Elektron-Neutrinos, Myon-Neutrinos und Tauon-Neutrinos, die vermutlich fluktuieren und sich ineinander umwandeln können, dazu vielleicht noch Antiteilchen (falls sie nicht ihre eigenen Antiteilchen sind), und nachweisen lässt sich wohl nur eine der drei Sorten. Es ist also unglaublich schwierig. Ein fehlender Nachweis für kalte Dunkle Materie heißt also nicht, dass es sie nicht gibt. Womöglich hat man nur nicht an der richtigen Stelle genau genug hingeschaut. Und wenn statt der dicken WIMPs, die so massiv wie schwere Atomkerne sein müssten, superleichte Axionen die Dunkle Materie darstellen, ist der Nachweis gleich noch einmal die eine oder andere Größenordnung schwieriger. Also, es ist hinreichend nachgewiesen, dass es Teilchen mit den gewünschten Eigenschaften gibt, die sich dadurch prinzipiell schlecht detektieren lassen. Der Nachweis, ob es in der Realität ausreichend viele davon gibt und das mit der “richtigen” Geschwindigkeit, lässt natürlich auf sich warten.

    Und was die Widersprüche zwischen Quanten- und Relativitätstheorie angeht, so gilt die ja nicht generell. Die eine beschreibt vieles im sehr kleinen, die vieles im sehr großen. Die Erklärung, wieso etwa Gold diese ungewöhnliche Farbe hat, fällt ganz bestimmt in den Bereich der Quantentheorie, trotzdem erklärt man sie sich heute mit relativistischen Elektronen. Mit anderen Worten: Die Relativitätstheorie erklärt einen Effekt, für den sie eigentlich gar nicht gemacht worden war! Überhaupt wurde Albert Einstein als der Vater der Relativitätstheorien und ein von vielen Vätern der Quantentheorie links und rechts überholt. Auch wenn er für die Quantentheorie einige Grundlagen gelegt hat, hat ihm die Richtung, die sich dann eingeschlagen hat, überhaupt nicht gefallen. Dumm für ihn war nur, dass sie an vielen Stellen sehr gut funktioniert und dass seine Versuche, sie zu widerlegen, allesamt gescheitert sind. Genau so ist ihm auch seine auf den Relativitätstheorien aufbauende Kosmologie aus den Händen genommen wurde. Wollte Einstein noch am statischen Universum festhalten, haben die Kollegen seine Ideen benutzt, das Gegenteil zu belegen.

    Bekanntlich geben Physiker bei ihren Rechnungen oftmals (oder immer?) auch eine Fehlerabschätzung an. Mit anderen Worten: Einem Physiker ist es durchaus klar, dass er sich immer irrt. Der Trick ist halt, sich eben ziemlich gut zu irren.

    Oder wie ich des öfteren zu sagen pflege: Die Physik zeigt uns, dass das Universum ein ziemlich durchgeknallter Ort ist.

  34. #34 Frantischek
    23. Mai 2017

    Die Dunkle Materie wurde nicht nachgewiesen, auch wenn man viele Hinweise darauf gefunden hat, dass es sie geben muss.

    Das würde ich ändern auf:
    “..auch wenn man viele Hinweise gefunden hat, dass es da ETWAS geben muss das die Effekte hervorruft.”

    Im Grunde könnte das doch auch etwas ganz anderes sein als Materie. Sogar etwas was so neu und weird ist, dass wir überhaupt noch keine Vorstellung davon haben können.
    Ist zwar sehr unwahrscheinlich, aber nicht unmöglich…

    @Ambivalent:

    Dunkle Materie wäre nämlich allgemein dünn verteilt, herkömmliche Materie dagegen “verklumpt” zu Sternen und anderen Objekten, die sich dort, wo sie sind, stärker bemerkbar machen als amderswo. Selbst wenn es nicht nur ein oder zwei Universen sind, die auf unseres einwirken, sondern viele, müsste man die “Schatten” von Sternen in anderen Universen immer noch messen können. Bis jetzt ist das aber nicht geschehen.

    Warum? Wenn es sehr viele Nachbaruniversen gäbe, und deren jeweilige Gravitation nur sehr schwach auf die anderen wirken würde, wäre das genau das Bild das ich erwarten würde.

    Das jedes Universum einen “dunklen” Anteil Gravitation spürt der durch die unzähligen aber sehr schwachen Quellen, die sich nur ungefähr an den gleichen Positionen befinden, gleichmäßig “verschmiert” erscheint.

    Ich denke dass das ab einer gewissen Menge an Nachbarn nicht mehr unterscheidbar, oder zumindest außerhalb unserer Messgenauigkeit wäre.

    Wie genau können wir denn momentan DM Wolken um Galaxien vermessen? Die Bilder die ich kenne zeigen ja Wolken in der Größenordnung von Galaxieclustern.
    Ich kann mir nicht vorstellen dass man da Dichteschwankungen in der Größenordnung von Sternen oder Verschiebungen von Galaxien (in den andern Universen) um ein paar hundert Lichtjahre erkennt.

    Falls ich mich irre bitte ich um Aufklärung. 😀

  35. #35 Bullet
    23. Mai 2017

    Ich geb mal kurz meine zwei Cent der Auffälligkeiten ab, nachdem ich Christophers Texte gelesen hab:
    1)

    man traut sich kaum irgendwo mal eigene Überlegungen zu diskutieren, weil man ganz schnell in der Ecke der “exotischen” Theorien ist die sowieso belächelt werden. Vor allem wenn man wie ich unangenehme Fragen stellt.

    Mit Verlaub: nimm dich mal nicht so wichtig. Was sollen “unangenehme Fragen” sein? In der Physik? Jede Frage, die altbekannte Routinen aus dem Gleichgewicht bringt, ist eine gute, wichtige und sehr förderliche Frage. Und als solche garantiert nicht “unangenehm”. Unangenehm sind die Leute, die alle fünf Minuten glauben, Einstein widerlegt zu haben, weil sie im Gedankenexperiment ein Raumschiff 1 Jahr lang mit 1g beschleunigt haben. Und genau in deren Welt paßt deine Verwendung dieses Wortes.

    2)

    Bisher gibt es keinen Nachweis für dunkle Materie genauso wenig wie für dunkle Energie. Man war bisher zu keinem Zeitpunkt in der Lage auch nur ein einziges dunkle Materie Teilchen zu detektieren.

    Ich dachte, du studierst Physik? Okay, Physiker haben es manchmal nicht so mit Rechtschreibung (*hüstel*), aber wie man Teilchen detektiert, soltest du wissen. Was war nochmal das Merkmal für Dunkle Materie? Ach ja: keine elektromagnetische Wechselwirkung. Na dann detektier mal. Solltest du wissen als Physikstudent.

    3)

    die Physik [hat] ein Problem und zwar genau dieses, dass wir etwas Grundlegendes nicht verstanden haben. Der Beweis dafür ist übrigens, dass die Quantenmechanik nicht mit der Relativitätstheorie in Einklang zu bringen ist.

    Ist das ein Beweis? Physikstudenten sollten doch Mathe können … also hol ich dich da mal ab.
    f(x)=1/x. Klar soweit?
    Für x (+ ->0) ist lim f(x) = +∞,
    für x (- ->0) ist lim f(x) = -∞.
    Für x=0 bekommst du daher zwei verschiedene Grenzwerte, je nachdem, von wo aus du deine Grenzwertbetrachtung startest. Und mal echt jetzt: weiter ausenander als +∞ und -∞ können wohl keine Ergebnisse sein, oder?
    Obwohl also die beiden Teilgraphen dieser Funktion nichtestens in Einklang gebracht werden können, ist das kein “Beweis” dafür, daß die Funktion f(x) = 1/x nicht existiert oder falsch ist.
    Wir beide haben keine Ahnung, warum sich QM und RT scheinbar nicht unter einen Hut bringen lassen. Darin aber einen “Beweis” für irgendwas zu sehen, ist bestenfalls kurzsichtig.

    4)

    Das Universum ist sehr flach. Gesetzt der Fall beim Urknall sind mehrere Universen entstanden die Flach übereinander liegen

    Du hast keine Ahnung, was hier “flach” bedeutet, oder? Das ist ein SO klarer Fall von auf-dem-falschen-Dampfer, daß es nur so zwitschert. Wundert mich ein wenig bei einem Physikstudenten. Welches Semester?

    5)

    Genauso ist es vermutlich der kosmologische Normalfall, dass es neben unserem Universum Milliarden andere gibt.

    Was war nochmal die Bedeutung des Wortes “Universum”?

    6)

    Mir ist natürlich bewusst, dass man die Teilchen erst mit einem Messgerät detektieren muss und dann ist das Resultat ein anderes als wenn man kein Messgerät dazwischen hätte. Das ändert ja nichts an der Tatsache, dass eine solche Theorie in meinen Augen völlig unlogisch ist.

    Was? Du studierst angeblich Physik und findest es unlogisch, daß die Messung einer Anzahl von Hühnereiern mittels eines von Hulk-in-Rage gehaltenen Baseballschlägers ein anderes Ergebnis zeigt als eine Messung mit Fotozelle? Ich muß doch sehr bitten. Das ist Stoff des ersten Semesters, und ich zumindest hab das in der Vorlesung gehabt.

    Nachsatz:

    In jedem Fall sind Teile der QM nicht schlüssig.

    Wie sag ichs jetzt… wenn Realität und deine Vorstellungen nicht übereinstimmen, wo liegt dann der Fehler?

  36. #36 Alderamin
    23. Mai 2017

    @Frieda Blum

    Wenn die These des Blasenmultiversums stimmt, in welchem es einen ewig inflationär expandierenden Raum gibt, in dem lokal immer wieder Blasen mit langsamerer Expansion entstehen und wachsen (wie unser Universum dann eine wäre), dann wäre möglicherweise in diesen Blasen der Zeitpfeil ein anderer als außerhalb, der Pfeil würde von der Außenseite der Blase nach innen zeigen. Denn der Außenrand der Blase wäre der Übergang von der inflationären Expansion zur langsameren, das wäre im jeweiligen Blasenuniversum der Urknall, der zur Zeit 0 stattfindet. Obwohl die Blase also aus Sicht des umgebenden Raums wächst und die Außenwand über einen langen Zeitraum existiert, existiert sie aus der Innensicht der Blase nur einen Moment lang; die Entropie ist an der Blasenhülle minimal und wird nach innen größer. Wenn der Zeitpfeil in Richtung wachsender Entropie zeigt, zeigt er also radial in die Blase hinein.

    Wenn dem so wäre, dann lägen andere Universen, die sich außerhalb unserer Universumsblase befinden, in unserer Vergangenheit, vor unserem Urknall.

    Wenn hingegen das “Ekpyrotische Universum” stimmt, dann gibt es parallele Branen in einer höheren Dimension, von denen unser Universum eine ist (eine mit drei Raum- und einer Zeitdimension). Die könnten ein paar cm voneinander entfernt in einer 4. Raumdimension schweben und beim Urknall zusammengestoßen sein.

    Dann gäbe es noch die Möglichkeit, dass andere Universen einfach sehr weit entfernt in unserer Raumzeit existieren und möglicherweise wegen der endlichen Zahl von Wiederholungsmöglichkeiten für die Quantenzahlen von einer bestimmten Menge von Teilchen das lokale Universum in allen Varianten unendlich oft spiegeln. Brian Greene spricht hier vom “Steppdeckenuniversum”.

    Daneben gibt es noch weitere denkbare Varianten. Ich empfehle dazu Brian Greenes Buch “Hidden Reality” (gibt’s auch in deutsch), das Florian hier schon ausführlich vorgestellt hat, siehe “Kategorien” auf dieser Seite hier, letzter Link.

  37. #37 Alderamin
    23. Mai 2017

    @Frantischek

    Wie genau können wir denn momentan DM Wolken um Galaxien vermessen? Die Bilder die ich kenne zeigen ja Wolken in der Größenordnung von Galaxieclustern.

    Heutzutage durch Computeranalyse der durch ihre Gravitation verzerrten Bilder von Hintergrundgalaxien. Man kann durchaus auch die Dunkle Materie um einzelne Galaxien ausmessen, wenn sie recht groß sind. Ansonsten schließt man über die Bewegungsgeschwindigkeit von Sternen innerhalb Galaxien auf deren Gesamtmasse und schätzt die sichtbare Materie anhand dessen, was leuchtet. Der Rest ist dann Dunkle Materie.

    Man kann aber auch auf die Dichte der Dunklen Materie in der Hintergrundstrahlung und bei der Entstehung der ersten Elemente schließen. Das passt alles zusammen. Und ohne die Dunkle Materie funktioniert keine Simulation der Galaxienentstehung. Da muss man etwas annehmen, das sich wie ein ideales Gas verhält und 4/5 der Gesamtmasse ausmacht. Im Prinzip könnten das auch primordiale Schwarze Löcher sein. Über die Art und Größe der Teilchen ist nichts bekannt, nur ihre Dichte im Raum und dass sie verhältnismäßig langsam unterwegs sein müssen, sonst würde sie nicht in Clustern und Galaxien zusammen rücken.

  38. #38 Berlin
    23. Mai 2017

    Ich verstehe es nicht, heißt Universum nicht Universum weil es alles umfaßt, also sind die “anderen Universen” nicht Bestandteil des “Universums” also eine Strukturfrage?
    LG Bernhard

  39. #39 Captain E.
    23. Mai 2017

    @Berlin:

    Das kann man so sehen. Üblicherweise versteht man unter “Universum” aber all das, was aus jenem Urknall heraus entstanden ist, dem alles entstammt, was wir selber sind oder was wir sehen können. Sollte es etwas ähnliches geben, nur von uns völlig getrennt, also als Ergebnis eines anderen Urknalls, könnte man auch das “Universum” nennen.

  40. #40 Alderamin
    23. Mai 2017

    @Berlin

    Na ja, früher, als der Begriff geprägt wurde, war er auch so umfassend gemeint (die Erde mit den umgebenden Planeten- und der Fixsternsphäre). Erst seit ein paar Jahrzehnten gibt es überhaupt die Idee, dass es mehr als das gibt, was wir bisher als “Universum” kannten (das, was man sieht), da muss man die Semantik entsprechend anpassen. Oder neue Wörter erfinden (“Multiversum”).

    Aber manche meinen ja immer noch nur die Erde, wenn sie das Wort “Welt” benutzen. Oder gar nur ihr kleines Dorf.

  41. #41 Mirko
    HH
    23. Mai 2017

    Also jetzt hört doch bitte auf, auf Christopher rumzukloppen. Er hatte mE nichts weiter vor, als daran zu erinnern, dass man Alternativen denken dürfen muss.
    Neulich hat FF darüber geschrieben, dass er und andere Fachleute sich dauernd mit Schriften von Möchtegernexperten herumschlagen müssen, die eher wirre Theorien aufstellen und meinen, schlauer zu sein als die Gemeinde der ausgebildeten Astro- und Teilchenphysiker. Das kann ärgern. Christopher würde ich aber nach den Beiträgen oben nicht dazu zählen. Ich kenne ihn nicht, aber das trifft ja wohl auch auf alle anderen Kommentatoren zu?
    Ich denke, den meisten Lesern hier geht es so, dass sie schon mal über ‘schräge’ Antworten auf die aktuell offenen Fragen der Physik philosophiert haben. Anders zu denken, muss erlaubt sein. Theorien infrage zu stellen, muss erlaubt sein. Und ich deute FF’s Blog und die Schreibwettbewerbe so, dass er das auch pushen will.

  42. #42 AmbiValent
    23. Mai 2017

    @Mirko
    Es ist völlig in Ordnung oder sogar gut, wenn man Alternativen denkt. Das Problem entsteht dann, wenn man versucht, aus Argumenten gegen die momentan akzeptierte Lösung zu folgern, es gäbe gar keine Lösung, oder dass alle Ansätze gleichwertig wären.

    Will man eine zumindest gleich gute Lösung finden, muss diese zumindest gleich gute Vorhersagen machen wie die bestehende.

  43. #43 Bullet
    23. Mai 2017

    @Mirko:

    Christopher würde ich aber nach den Beiträgen oben nicht dazu zählen.

    Richtig, denn er hat ja keine eigenen seltsamen Hypothesen aufgestellt oder sie gar als “noch unverstandene Wahrheit” bejubelt.
    Jedoch ist nicht nur mir aufgefallen, daß seine “kritischen” Ansätze zur aktuellen Situation der physikalischen Modellierung der Kosmologie bedenklich häufig einem Muster folgen, das in nur hauchdünnem Abstand zur Denkblase genau jener Leute, die ihrer Privatphysik überlegenen Stellenwert zuordnen, herumlungert. Stichwort “versteh ich nicht, ist also unlogisch”.
    Und das muß man auch ansagen.
    Ich weiß nicht, wie lange du hier schon mitliest, aber hier sind Kommentatoren unterwegs, die seit 7 bis 8 Jahren mit Privatphysikern zu tun haben. Glaub mir: die Argumentations- und Logikmuster der Privatphysiker sind deutlich erkennbar und deutlich verschieden von dem, was man so rausläßt, wenn man zumindest ehrlich mit sich selbst ist und seinen eigenen Laienstatus akzeptiert.

  44. #44 Phero
    23. Mai 2017

    @Mirko: Weiterhin ist das beständige Erinnern daran, dass man Alternativen denken dürfen muss, auch recht unnötig. Die allermeisten Wissenschaftler werden sich dessen bewusst sein. Allerdings gebe ich zu, dass dies möglicherweise nicht immer so herüberkommt, vor allem durch Wissenschaftssendungen und ähnliches, die häufig einen extremen Schwerpunkt auf “Neues” legen – so, als ob man dies besonders hervorheben muss und gleichzeitig häufig einen Konflikt beschwört, der sich tatsächlich häufig danach anhört, als seien alle anderen Wissenschaftler verbohrt und engstirnig (bei Gelegenheit wäre das ein interessantes Projekt, diese Wissenschaftsvermittlung zu erforschen).

    Ich finde auch nicht, dass auf ihm herumgekloppt wurde – da gab es schon weitaus schlimmeres. Aber man darf nicht denken, dass auf Aussagen kein Widerspruch kommt. Und eine Aussage wie (nicht wörtlich zitiert) “Seid doch mal ein bisschen offener und lasst neue Ideen zu” regt einfach zu Widerspruch an, weil annimmt, dass das Gegenteilt der Fall sei. Und das stimmt einfach nicht.

  45. #45 AmbiValent
    23. Mai 2017

    Ein Beispiel, wie es gehen kann, wäre die Suche nach “Planet Neun”: Motiviert von unerwarteten Eigenschaften der Asteroidenbahnen jenseits des Neptun wurden immer wieder Arbeiten publiziert, in denen ein (mindestens marsgroßer) Körper dafür verantwortlich gemacht wurde, und die Eigenschaften dieses Körpers (Masse, Bahn) vorhergesagt.

    Aber nicht nur wurden diese Körper bis jetzt nicht gefunden, die meisten Ansätze wurden sogar in anderen Arbeiten widerlegt. Das heißt nicht, dass man gar nicht erst versuchen sollte, einen solchen Körper vorherzusagen. Denn inzwischen gibt es eine Vorhersage (von Brown und Batygin), die nicht nur die Bahnen der Asteroiden erklärt, sondern auch die Abweichung der Rotationsachse des Sonnensystems von der der Sonne (und dies war gar nicht die ursprüngliche Absicht von Brown und Batygin gewesen).

    Aber genauso sehr, wie man sich von vielen vorhergehenden Widerlegungen anderer Vorschläge nicht abschrecken lassen sollten, so sehr sollte man auch im Blick behalten, dass die meisten der Vorschläge eben falsch waren, und dass man deshalb auch nachweisen muss, dass der eigene Vorschlag besser ist und zu einem echten Nachweis führt.

  46. #46 Steffmann
    23. Mai 2017

    Das Multiversum ergibt sich mathematisch plausibel aus der Inflation, oder nicht ? Die Inflation stellt kaum ein Kosmologe noch in Frage ? So what ?

  47. #47 Mike
    24. Mai 2017

    Kleines Detail:
    Wenn die Chance 2% beträgt, kannst du nicht einfach sagen, dass du [zwingend] ein Universum mit kaltem Fleck hättest, wenn du 50 Stück davon entstehen lässt. So funktioniert Wahrscheinlichkeitsrechnung nicht 🙂

  48. #48 Alderamin
    24. Mai 2017

    @Steffmann

    Nicht bei allen Varianten der Inflation muss ein Multiversum entstehn, es gibt auch die Möglichkeit eines “slow roll” vom inflationären Vakuum zu gewöhnlichen, der sich überall gleichzeitig ereignet haben könnte. Außerdem ist die Inflation noch nicht durch den Nachweis der charakteristischen “B-Moden” in der Polarisation der Hintergrundstrahlung belegt. Bisher ist sie also nur eine gute Idee mit mehreren möglichen Varianten.

  49. #49 Florian Freistetter
    24. Mai 2017

    @Mike: Das mit den 1:50 steht genau so im paper/dee Pressemitteilung. Und ich denke eigentlich nicht dass das falsch ist.

  50. #50 Frieda Blum
    24. Mai 2017

    Blasenmultiversums
    Wenn dem so wäre, dann lägen andere Universen, die sich außerhalb unserer Universumsblase befinden, in unserer Vergangenheit, vor unserem Urknall.

    Ist es bei einem Blasenmultiversums vorstellbar, Informationen von einem zum anderen Universum zu erhalten? Jedes Universum befindet sich doch in seiner eigenen Zeitblase, oder?

  51. #51 Frantischek
    24. Mai 2017

    @Florian:

    Wenn man den Wahrscheinlichkeitsbegriff so verwendet hat man aber ein Problem:

    Woher “wissen” 49 von 50 Universen dass es bereits eines mit kaltem Fleck gibt, und sie deshalb keinen entwickeln sollen? (Ich weiß dass das sehr blöd ausgedrückt ist, aber besser kann ichs nicht.)

    Ist es in der Realität nicht eher so dass die Wahrscheinlichkeit bei jedem einzelnen Fall neu berechnet werden muss, und entweder 0 oder 1 beträgt? Je nachdem ob das Ereignis eintritt oder nicht?

  52. #52 Alderamin
    24. Mai 2017

    @Frieda Blum

    Es sollte nicht möglich sein, Information durch die Feuerwand zu schleusen, die eine Blase umgibt, denn dabei geht sämtliche Materie, ja sogar das Vakuum kaputt. Es werden gewissermaßen neue Naturkonstanten ausgewürfelt und alle Teilchen entstehen neu (falls sie bei den erwürfelten Konstanten entstehen können).

    So wird das üblicherweise populärwissemschaftlich dargestellt. Was am Ende heraus kommt, ist dem Zufall überlassen.

  53. #53 Boombox
    24. Mai 2017

    Eine Wahrscheinlichkeit von 1/50 (das wäre eigentlich 1:49 und nicht 1:50, aber vielleicht sind die englischen Sprachkonventionen da anders) bedeutet nicht, dass bei genau 50 Versuchen genau ein Universum einen kalten Fleck hat.

    Bei einem Würfel beträgt die Wahrscheinlichkeit für jede Zahl 1/6, das bedeutet aber selbst bei einem perfekten Würfel nicht, dass bei genau 6 Würfen jede Zahl genau einmal gewürfelt wird. Es kann auch sechsmal die gleiche sein oder dreimal die gleiche und noch 3 andere Zahlen jeweils einmal oder jede andere mögliche Kombination für 6 Zahlen, von denen jede unabhängig von denen der anderen einen Wert zwischen 1 und 6 haben kann (ganze Zahlen meine ich natürlich nur). Erst wenn die Anzahl der Würfe gegen unendlich strebt, nähert sich die Häufigkeit der einzelnen Zahlen jeweils 1/6 an.

  54. #54 Boombox
    24. Mai 2017

    @Frieda Blum: Das dürfte schon deshalb nicht funktionieren, weil aus Sicht jedes Universums die anderen in dessen Vergangenheit liegen, wie es Alderamin schreibt, selbst wenn da keine Feuerwand wäre. In die Vergangenheit kann man meines Wissens (bisher?) keine Informationen schicken.

  55. #55 Phero
    24. Mai 2017

    @Mike/Frantischek:
    Schon klar. Ist aber eine häufige und bequeme Art sich auszudrücken und die Leute, die sich ein wenig mit Wahrscheinlichkeitsrechnung auskennen, wissen natürlich, dass das statistisch gemeint ist. Streng durchgerechnet beträgt die Chance bei 50 Universum 73,6%, dass mindestens ein Universum solch einen Fleck hat.

  56. #56 Alderamin
    24. Mai 2017

    @Boombox

    Auch wieder wahr. Andere Universen wären in unserer Vergangenheit – und unseres in deren! Theoretisch könnte nur Information aus dem inflationären Raum in die Blase hinein dringen, aber nichts hinaus. Aber wegen der Feuerwand geht eben nicht einmal das.

    Es gibt allerdings die nette Hypothese, dass auch innerhalb Schwarzer Löcher wieder neue Universen entstehen könnten, die die Naturgesetze des Universums des Schwarzen Lochs “erben”. So würden Universen, die Schwarze Löcher hervorbringen können, wieder neue Universen erzeugen, die das auch können.

  57. #57 Captain E.
    24. Mai 2017

    @Alderamin:

    Und das “Erben” erfolgte womöglich mit Mutationen, oder? Zumindest müssten die Dimensionen wohl anders aussehen als in “unserem” Universum, und das könnte sich auch auf die Zeit beziehen. Man wage sich aber gar nicht vorzustellen, was ein “Verdampfen” eines Schwarzen Lochs (durch Hawking-Strahlung) für das Universum im Inneren bedeutete.

  58. #58 Phero
    24. Mai 2017

    @Captain E.: Jap, das ist die Idee. Tatsächlich würde das bedeuten, dass es einen evolutionären Prozess gibt, der das Entstehen lebensfreundlicher Universen begünstigst (zumindest die Bedingungen für stellare schwarze Löcher sind ähnlich wie diejenigen für die Entstehung von Leben).

    Das Verdampfen kann alles mögliche bedeuten – das Tochteruniversum könnte zerfallen. Vielleicht wird es aber auch nur sozusagen “abgekoppelt”.

  59. #59 Alderamin
    24. Mai 2017

    @Captain E.

    Es soll seinen eigenen Raum erzeugen und sich abkoppeln. Das Verdamofen wird ihm egal sein.

  60. #60 Frieda Blum
    24. Mai 2017

    Keine Informationen von einer Blase zur anderen heißt doch auch, wir bekommen absolut nichts mit von der anderen Blase. Also kann es auch keine Hinweise für so etwas geben. Oder anders ausgedrückt, falls wir hier eine Anomalie detektieren kann es nicht von so einem Universum stammen. Stimmt doch, oder?

  61. #61 Boombox
    24. Mai 2017

    @Frieda Blum: Du meinst so etwas wieden ungewöhnlich kalten Fleck in der Hintergrundstrahlung (falls der doch von der Wechselwirkung mit einem anderen Universum stammt)?

    Soweit ich weiß, sollen in einem Blasenmultiversum gerade im Entstehen begriffene Universen kollidieren können, solange der inflationär expandierende Raum zwischen ihnen noch nicht zu groß ist. Und das würde sich dann durch entsprechende Strukturen in der Hintergrundstrahlung (die ja aus den Anfangszeiten des Universums stammt) äußern.

  62. #62 Frieda Blum
    24. Mai 2017

    @Boombox

    Das verstehe ich nicht, als unser Universum im Entstehen war, waren die anderen Universen vergangen, oder?

  63. #63 Frantischek
    24. Mai 2017

    Für uns, ja.
    Für den ewig inflationären “Raum” in denen wir uns bewegen/entstehen, aber nicht.
    Bitte korrigieren, falls Blödsinn!

    @Captain, Alderamin, Boombox:

    Das mit den Universen aus SL, und vor allem die Idee mit der Evolution dabei.
    Könnt ihr mir da Lesetipps, oder Links, geben?

    Dangöööö!

  64. #64 Boombox
    24. Mai 2017

    @Frieda Blum: Wie Alderamin weiter oben schon geschrieben hat, wäre in den verschiedenen Universen und im Raum dazwischen der Zeitpfeil unterschiedlich. In den einzelnen Universen wäre die Gegenwart in der Mitte und der Urknall ganz außen am Rand der Blase. Von außen betrachtet wäre die Zeit in einem der Universen also quasi wie eine weitere Raumdimension und um in den Raum zwischen den Universen (oder gar in eines der anderen Universen) zu gelangen, müsste man in Richtung Vergangenheit bis über den Urknall hinaus reisen. Es kommt also darauf an, von wo aus man es betrachtet. In einem der Universum (wie unserem) haben die anderen in der Vergangenheit vor dem Urknall existiert, von außen betrachtet existieren sie parallel zueinander.

  65. #65 Boombox
    24. Mai 2017

    @Frantischek: Ich bin mir leider nicht mehr ganz sicher, aber ich glaube, Brian Greene schreibt in seinem Buch “Der Stoff, aus dem der Kosmos ist” auch darüber. Ist aber auch ansonsten ein sehr empfehlenswertes Buch. 🙂

  66. #66 Wizzy
    24. Mai 2017

    @Phero 63,6%

  67. #67 Phero
    24. Mai 2017

    @Wizzy: Stimmt – Subtrahieren ist aber auch kompliziert.

  68. #68 tomtoo
    24. Mai 2017

    @Boombox

    Wär ja auch denkbar, das es es eine Person auf der Erde gibt die immer nur sechsen würfelt ?

  69. #69 Alderamin
    24. Mai 2017

    @Frantischek

    Ich hatte die Geschichte mit den sich reproduzierenden Universen schon früher mal irgendwo aufgeschnappt und neulich bei diesem Autor wiedergefunden:

    https://arxiv.org/pdf/1305.4524.pdf

  70. #70 Rince
    24. Mai 2017

    Ohne mir jetzt alle Kommentare bis zum Ende durchzulesen möchte ich eine kleine Anmerkung zu diesem Artikel machen. Im Spektrum der Wissenschaft (4.17) gibt es einen Beitrag genau zu diesem Thema (Titel: Die große Leere.) Dort wird in kürze beschrieben dass in Richtung Kalter Fleck der Hintergrundstrahlung doch ein Supervoid gefunden wurde mit den Ausmaßen von 1,8 Millijarden Lichtjahre. Falls jemanden diesen Artikel interessiert schlag ich vor dort nachzulesen.

    Lg
    Rince

  71. #71 tomtoo
    24. Mai 2017

    @Rince

    Ok, ich die Motzbacke. Aber hättest du doch auch gleich verlinken können oder ?
    ; )

  72. #72 Steffmann
    25. Mai 2017

    @Alderamin #48:

    Danke für die Antwort. Dann hoffen wir mal, dass die B-Moden möglichst bald gefunden werden. Die Plausibiltät des Modells der ewigen Inflation ist imho unschlagbar. Keine blöden Diskussionen mehr über die unglaublich genaue Verteilung der Grundkräfte in unserem Universum. Keine pseudo-wissenschaftliche Diskussion mehr mit Gläubigen, wie bspw. hier:
    https://www.soulsaver.de/article/warum-gott-der-schopfer-ist/

  73. #73 Steffmann
    25. Mai 2017

    Wenn ich mich recht erinnere, dürfte der Nachweis von B-Moden so ziemlich dem von der Entdeckung ausserirdischem Lebens entsprechen. Also extrem aufwändig und schwer zu realisieren. Maybe, dass das zukünftig etwas leichter wird, wenn man in ferner Zukunft Detektor-Kollaborationen (Ligo) ins Weltall hiefen und dort zuverlässig zum Arbeiten bringen könnte. Freilich zweifle ich am Interesse daran. Denn letztlich würde ein zweifelsfreier Nachweis eines Multiversums so einiges bewegen. Ähnlich wie es damals das heliozentrische Weltbild bewirkte. Aber kann auch sein, dass ich mich gerade verrenne hier….

  74. #74 Alderamin
    25. Mai 2017

    @Steffmann

    LIGO (auf der Erde) bzw. eLISA (im All) weisen aktuell umherschwirrende Gravitationswellen nach. Die B-Moden sind nur zirkulare Polarisationsmuster in der Hintergrundstrahlung, die im Augenblick des Urknalls durch Gravitationswellen entstanden und im 380000 Jahre späteren Feuerball der Hintergrundstrahlung noch als Muster verblieben sind. Prinzipiell sind die B-Moden so schwer nicht nachzuweisen, sie sollten in den Daten der PLANCK-Sonde enthalten sein und das BICEP2-Projekt am Südpol sucht auch nach ihnen und hatte bereits eine Detektion vermeldet, die dann jedoch keine war. Offenbar ist die Signatur jedoch sehr schwach, oder sie fehlt ganz, oder der Staub der Milchstraße überdeckt sie zu sehr. Vielleicht kitzelt man sie ja bald doch noch aus den Messungen heraus.

  75. #75 Frieda Blum
    25. Mai 2017

    Ich glaube jetzt habe ich diese These halbwegs verstanden.
    Also am Anfang, als alles noch in der Singularität war gab es quasi eine Verbindung, später aber nicht mehr.

    Freilich zweifle ich am Interesse daran. Denn letztlich würde ein zweifelsfreier Nachweis eines Multiversums so einiges bewegen. Ähnlich wie es damals das heliozentrische Weltbild bewirkte.

    Was sollte es denn gravierendes bewirken? Das das ganze noch etwas komplizierter ist als bisher gedacht? Das wir noch kleiner sind als bisher gedacht?
    Ja und? Wenn es stimmen sollte, dann geht davon evtl. das Weltbild unter, nicht aber die Welt. Wäre nicht das erste mal dass man das Weltbild korrigiert hat.

  76. #76 Frantischek
    25. Mai 2017

    …bzw. eLISA (im All) weisen aktuell umherschwirrende Gravitationswellen nach.

    eLISA startet frühestens 2034.
    LISA Pathfinder ist schon im All.
    Soll aber bald beendet werden (oder wurde es schon, weiß ich nicht genau) und misst auch noch keine Gravitationswellen, wenn ich mich nicht sehr irre…

  77. #77 Alderamin
    25. Mai 2017

    @Frantischek

    Weiß ich. Das “aktuell” bezog sich nicht auf die aktuellen Teleskope, sondern auf die aktuell umherschwirrenden Gravitationswellen, wobei ich auch 2034 als “aktuell” bezeichne – im Vergleich zu den primordialen Gravitationswellen, die ihre Signatur in der Hintergrundstrahlung hinterlassen haben sollen. Sorry, war missverständlich formuliert.

  78. #78 Rudi
    Deutschland
    25. Mai 2017

    Lieber Florian,
    Ich folge deinem Blog jetzt schon ziemlich lange und muss dir für all das Wissen, das du damit vermittelst, danken. Ich bin kein Physiker, sondern lediglich Elektroingenieur, daher komme ich hin und wieder ins Stocken. Liegt einfach an fehlenden Grundlagen…
    Nach dem Lesen deines Beitrags versuchte ich, den Inhalt nachzuvollziehen, was mir jedoch nicht Recht gelang. Ich musste also pfuschen und mich im Nachgang über die gravitative Rotverschiebung informieren. Kannte ich noch nicht. Nun ist mir auch aufgefallen, warum ich deinen Beitrag nicht verstanden habe:
    Du schreibst, dass Licht beim Eintreten in ein Gravitationsfeld​ Energie verliert und sie beim Verlassen zurück erhält. Aber eigentlich ist es genau anders herum. Beim Eintreten wird die Frequenz des Lichts durch die Dilatation der Zeit scheinbar erhöht, was einer Erhöhung der Energie entspricht. Bis zum Austritt aus dem Feld wurde dieses durch die Expansion des Universums aber geschwächt, wodurch die scheinbare​ Reduzierung der Frequenz beim Verlassen des Feldes geringer ausfällt. Die Energie ist also gestiegen. In deinem Beitrag, den du verlinkt hast, schreibst du das auch so. Nur hier ist es scheinbar etwas durcheinander geraten…
    Das wollte ich nur loswerden, falls noch ein anderer Nichtphysiker stolpert.
    Rudi

  79. #79 Florian Freistetter
    25. Mai 2017

    @Rudi: “Nur hier ist es scheinbar etwas durcheinander geraten…”

    Ich bin gerade unterwegs und kann das nicht im Detail nachlesen. Aber im verlinkten Artikel beschreibe ich den Sachs-Wolfe-Effekt der beschreibt was passiert wenn das Licht durch ein starkes Gravitationsfeld fliegt. Hier geht es um den umgekehrten Effekt: Licht bewegt sich durch Regionen wo gar nix ist.

    (Falls das nicht der Grund für die Verwirrung ist sagt bitte Bescheid dann schau ich mir das in Ruhe an wenn ich morgen wieder zuhause bin und korrigiere im Fall eines Fehlers)

  80. #80 bruno
    25. Mai 2017

    @Rudi
    ..bin nicht so bewandert wie die meisten hier – aber die Wiki schreibt doch:

    Photonen, die von einer gravitativen Masse aufsteigen, werden weniger energiereich..

    Evtl. bringst du was mit Frequenz und Energie durcheinander…
    Als Analogie darfst du dir einen Hügel vorstellen, über den das Photon drüber muss. Auf dem Weg hinauf verliert es Energie – die es auf dem Weg hinunter wieder 1:1 gewönne – nur hat sich in der Zwischen-Zeit der Weg hinab verlängert und es gewinnt weniger Energie hinzu als es beim Weg hinauf verloren hat.
    Es ist hinterher weniger energiereich -> also längere Wellenlänge -> also roter als vorher.
    Wiki sagt zu Sachs-Wolfe:

    Während der Propagation der Photonen durch das Universum treffen sie weiter auf die Anisotropien der baryonischen Materie. Im Fall eines statischen Universums würden die Photonen aufgrund der Energieerhaltung beim Verlassen einer Anisotropie dieselbe Energie wieder aufnehmen, die sie beim Eintritt in die Anisotropie abgegeben haben. Da sich das Universum in der Zeit jedoch ausgedehnt hat, flacht das Gravitationspotential ab..

    (Das Gleiche also mit schöneren Worten)

    Und in FFs verlinktem Artikel steht es genau so:

    Anstatt eines Hügels läuft es jetzt [im Void] quasi durch ein Tal und beim Austritt aus dem Tal gewinnt es die Energie wieder zurück. Aber nicht, wenn das All expandiert und das Tal dabei gestreckt und flacher wird. Dann hat das Licht danach weniger Energie…

    Hier auch mal auf Alpha Centauri 186 (Sachs-Wolfe) und 181 (Silk-Dämpfung) hingewiesen – vielleicht hilfts.

    Und – sei mir nicht böse – Ingenieure und Physikstudenten sorgen hier aktuell ohnehin für Erheiterung…
    😉

  81. #81 Rince
    25. Mai 2017

    Hallo,

    das Spektrum der Wissenschaft muss man kauflich erwerben. Ich wollte hier keine Werbung machen.

    Lg
    Rince

  82. #82 Rudi
    26. Mai 2017

    @Florian
    “Seit Albert Einstein wissen wir, dass Licht von Gravitation beeinflusst wird: Es verliert Energie, wird also rötlicher, wenn es sich durch ein starkes Gravitationsfeld bewegt und kann diese Energie wieder zurück gewinnen, wenn es das Gravitationsfeld verlässt.”
    So erwähnst du hier im Artikel kurz den Sachs-Wolfe-Effekt, also falsch herum, wenn ich es richtig verstanden habe.
    Die Theorie der Voids geht ja auch nur von einem kontinuierlichen Sachs-Wolfe-Effekt im gesamten Universum aus, der in den Voids reduziert auftritt.

  83. #83 Steffmann
    28. Mai 2017

    @Frieda Blum:

    Was sollte es denn gravierendes bewirken? Das das ganze noch etwas komplizierter ist als bisher gedacht? Das wir noch kleiner sind als bisher gedacht?
    Ja und? Wenn es stimmen sollte, dann geht davon evtl. das Weltbild unter, nicht aber die Welt. Wäre nicht das erste mal dass man das Weltbild korrigiert hat.

    Der Vollständigkeit halber, wenn auch viel zu spät: Der Nachweis eines Multiversums würde jede “Gott-Theorie” ad acta legen. Es würde beweisen, dass wir ein reines Zufallsprodukt im unendlichen Produktsprozess der Inflation wären. Vor allem der katholischen Kirche würde ihr letztes Argument (Verteilung der Naturkräfte) dann auch noch entzogen.

    Und im Gegenteil, es würde nicht komplizierter, sondern einfacher. Natürlich nicht für Physiker, die würden sich die Haare raufen, da die es nun Bereich geben würde, die definitiv nie untersuchbar sein werden. Aber für den Normalsterblichen eben schon.

    Letztlich muss ich aber zugeben, dass, selbst wenn viel dafür spräche, und ein guter Nachweis gefunden werden würde, es an der Validierung scheitern würde. Es bliebe eine Hypothese. Die dunkle Materie konnte beispielsweise über den Gravitationslinseneffekt nachgewiesen werden. Das geht bei Paralleluniversen natürlich nicht.

  84. #84 Wizzy
    29. Mai 2017

    @Rudi #82
    Du liegst richtig: Licht gewinnt Energie, wenn es in ein Gravitationsfeld eintritt. “gravitational redshift” tritt beim Verlassen eines Gravitationsfeldes auf.
    Oben hatte das Florian verwechselt. Allerdings ist das noch nicht der Sachs-Wolfe-Effekt, sondern soll (sobald korrigiert) als Grundlage zu dessen Verständnis dienen.

  85. #85 Florian Freistetter
    29. Mai 2017

    @Wizzy: “Oben hatte das Florian verwechselt.Allerdings ist das noch nicht der Sachs-Wolfe-Effekt

    Ich wollte allerdings auch nicht erklären was Licht macht wenn es in ein Gravitationsfeld eintritt. Sondern was passiert wenn es eine Void durchquert (das Gegenteil eines Gravitationsfeldes).

  86. #86 Frantischek
    29. Mai 2017

    Der Nachweis eines Multiversums würde jede “Gott-Theorie” ad acta legen.

    Von wegen! Hab erst vor ein paar Tagen auf irgendeiner Christenseite gelesen dass Gott NATÜRLICH auch unendlich viele Universen erschaffen kann.

  87. #87 tomtoo
    29. Mai 2017

    @Frantischek

    Uiii. Stressiger Job wenn er für eine Erde schon sechs Tage braucht und einen um sein Werk zu bewundern…. ; )

  88. #88 Physik-Fan
    29. Mai 2017

    @Phero #13
    Es konnten zwar noch keine dunklen Materieteilchen detektiert werden, aber das heißt nicht, dass es keinen Nachweis gibt. Indirekte Hinweise können ebenso gut wie direkte sein und die indirekten Hinweise auf Dunkle Materie sind gewaltig. Keine andere bisher vorgeschlagene Hypothese kann diese Phänomene alleine erklären

    Es gibt aber auch Befunde, die für eine modifizierte Grav. sprechen (s. „Gegenwind für die Dunkle Materie”, Spektrum d.W. 5.17). Eine Analyse von ca. 150 Galaxien ergab einen universellen Zusammenhang zwischen der Rotationsgeschw. bei einem bestimmten Radius und der innerhalb des Radius befindl. sichtbaren Materie. In der modifizierten Grav. wäre genau das zu erwarten.

    Das große Problem dieser Ansätze war ihr Ad-Hoc-Charakter. Warum weicht die Grav. von Newton/ART ab? Verlinde hat nun eine fundierte theoretische Begründung dafür geliefert und zwar auf Basis der neueren holographischen Ansätze, wo die Raumzeit als von Qubits aufgebaut betrachtet wird. Dem fügte er Dunkle Energie hinzu, modelliert als thermischen Effekt, und es kommt ein Verlauf der Grav. heraus, wo sie genau dort abzuweichen beginnt, wo es auch gemäß MOND der Fall wäre. Desweitern hat Verlinde aufgezeigt, dass sein Ansatz geeignet ist, die bekannten Differenzen zwischen der sichtbaren Materieverteilung und den indirekten Hinweisen auf weitere Grav.wirkungen (aus denen man ja auf die Anwesenheit von DM schließt) zu erklären, Beispiel: Bullet Cluster. Ein wesentlicher Punkt dabei ist die relative Unabhängigkeit von normaler Materie und Dunkler Energie und ihrer Grav.wirkungen. Verlindes Theorie wird als so gut betrachtet, dass auch Kosmologen, die den Modifikationsansätzen kritisch ggüb, standen, das als ernsthafte Alternative betrachten.

    Generell ist es nun mal Fakt, dass sich trotz jahrzehntelanger Suche keine irgendwie geartetete DM gezeigt hat. Alle Experimente waren negativ und die theoretischen Ansätze haben sich nicht erhärtet (z.B. SUSY, Axion). So lange sich nichts daran ändert, hängt die DM-Theorie in der Luft. Indirekte Hinweise können direkte Nachweise nie ersetzen. Man muss herausfinden, was die DM ist und dass sie sich passend zu den Befunden verhält. Nur so wird ein Schuh draus.

  89. #89 Physik-Fan
    29. Mai 2017

    @Steffmann #72
    Die Plausibiltät des Modells der ewigen Inflation ist imho unschlagbar.

    Das Inflationsmodell ist erklärungskräftig, gewisse Sachen ergeben sich ganz natürlich, aber leider hapert es mit Plausibilität, s. den Artikel ” Inflationsmodell in der Kritik“ von Steinhardt und Co-Autoren in der neuen Spektrum. Darin legen sie dessen Ungereimtheiten dar. Beim standardmäßig angenommenen einfachen Verlauf für die Inflationsenergiedichte müssten die Abweichungen von der Skaleninvarianz größer sein und sich Grav.wellen nachweisen lassen. Es bleibt nur eine Kurve, die zunächst ein Plateau aufweist und dann steil abfällt. Dabei muss aber das Inflatonfeld genau auf die jeweilige Energiedichte des Plateaus fallen. Das wäre ein extremer Zufall, “fast unmöglich” wie es im Artikel heißt. Zudem braucht das Feld etwas Zeit dazu, d.h. das Universum startet zunächst ohne Inflation. Das bedeutet, dass sich räumliche Inhomogenitäten entwickeln. Ab einer bestimmten Größe verhindern sie den Start der Inflation, denn dieser ist nur möglich wenn:

    1. Quantenfluktuationen keine große Rolle spielen und der Raum der ART genügt
    2. der Raum genügend flach ist.

    Die Wahrscheinlichkeit dafür, dass sich das irgendwo von alleine einstellt, ist nahe null. Außerdem setzt man genau das voraus, was erst das Ergebnis der Inflation sein sollte, sehr unbefriedigend.

    Die Autoren stellen das Inflationsmodell generell in Frage, aber dann stellt sich die Frage, wie Gleichförmigkeit und Flachheit zustande kommen. Ihre Lösung ist der “Urprall” statt dem Urknall. Der Expansion ist eine Phase der Kontraktion vorangegangen, wobei sich diese Zustände automatisch einstellen. Der Urprall tritt ein, bevor das Universum so sehr geschrumpft ist, dass Quanteneffekte relevant werden. Es bleibt im Gültigkeitsbereich der ART. Im Modell ist auch detailliert beschreiben, wie es von der Kontraktion zur Expansion kommen konnte.

    Der Vorteil des Modells ist, dass es auf wenigen Annahmen beruht, anders wie die Inflation und dass keine Myriaden von Universen impliziert werden, es bleibt bei einem. Allerdings, man fragt sich, ob und wie die Kontraktion experimentell nachgewiesen werden könnte. Auch die Feineinstellung der Naturkonstanten dürfte offen bleiben.

  90. #90 Steffmann
    29. Mai 2017

    @Physik-Fan:

    Das wäre ein extremer Zufall, “fast unmöglich” wie es im Artikel heißt.

    Bei unendlichen Versuchen, und darum geht es ja bei der ewigen Inflation, ist das unerheblich. Die prozentuale Verteilung der 4 (oder wie mancher meint, eigentlich nur 3) Naturkräfte, ist ja ähnlich im Zufallsbereich angesiedelt. Unser Universum ist eben unter Umständen nichts als das Produkt von unendlichem Versuch und Irrtum. Da spielen Wahrscheinlichkeiten keine Rolle.

  91. #91 Steffmann
    29. Mai 2017

    @Physik-Fan:

    Die Autoren stellen das Inflationsmodell generell in Frage, aber dann stellt sich die Frage, wie Gleichförmigkeit und Flachheit zustande kommen. Ihre Lösung ist der “Urprall” statt dem Urknall.

    Ockhams Razor.

  92. #92 Steffmann
    29. Mai 2017

    @Frantischek:

    Von wegen! Hab erst vor ein paar Tagen auf irgendeiner Christenseite gelesen dass Gott NATÜRLICH auch unendlich viele Universen erschaffen kann.

    Ja ne, iss klar. Die krallen sich alles, können sie ja auch, da Glauben NUR auf Behauptung basiert.

  93. #93 Alderamin
    30. Mai 2017

    @Physik-Fan

    Beim standardmäßig angenommenen einfachen Verlauf für die Inflationsenergiedichte müssten die Abweichungen von der Skaleninvarianz größer sein und sich Grav.wellen nachweisen lassen. Es bleibt nur eine Kurve, die zunächst ein Plateau aufweist und dann steil abfällt. Dabei muss aber das Inflatonfeld genau auf die jeweilige Energiedichte des Plateaus fallen.

    Wer kennt denn genau den Verlauf der Kurve des Inflatonfelds? Es ist ja nicht einmal klar, ob es das Higgs-Feld ist oder etwas anderes. Es gibt alle möglichen Varianten des Inflationsmodells.

    Zudem braucht das Feld etwas Zeit dazu, d.h. das Universum startet zunächst ohne Inflation. Das bedeutet, dass sich räumliche Inhomogenitäten entwickeln.

    Auch hier: es gibt da viele Möglichkeiten. Beim Ur-Inflationsmodell ging die Inflation los, nachdem die Temperatur so weit gefallen war, dass die starke und schwache Kernkraft sich trennten, das war schon nach 10-^-35 s der Fall. Das ist ziemlich kurz. Bei der ewigen Inflation kann diese schon beliebig lange im Gange sein und alle Inhomogenitäten längst ausgebügelt sind.

    Ihre Lösung ist der “Urprall” statt dem Urknall.

    Es gibt einen Haufen Varianten für die Entstehung des Universums. Erst wenn wir die Signatur von Gravitationswellen in der Hintergrundstrahlung gefunden haben, können wir aussieben, welche noch in Frage kommen. Wohlmöglich werden wir es nie herausfinden, welche die richtige ist.

  94. #94 Physik-Fan
    30. Mai 2017

    @Alderamin
    @Steffmann

    Ich bin kein Vertreter des Urpralls, sehe nur durch den Artikel, dass es ganz andere Ansätze gibt. Also locker die Inflation als gegeben zu betrachten, ist nicht. Bitte lest mal den Artikel durch und Kommentare am besten direkt an die Autoren 😉 ich zitiere ja nur. Der Mentor der Sache ist Paul J. Steinhardt, Inhaber des Einstein-Lehrstuhles in Princeton, also nominell einer der führenden Kosmologen. Man kann davon ausgehen, dass er über die Zusammenhänge so gut Bescheid weiß, wie wenige andere. Mit dem Artikel lehnt er sich weit aus dem Fenster, muss sich also seiner Sache recht sicher sein. Sein guter Ruf steht auf dem Spiel.

    Es mag zwar unterschiedlichste Verläufe der Inflation geben, aber gemäß dem Artikel sind es zwei Grundtypen, die herkömmliche Variante und das neuere Plateaumodell. Die Erstere bedeutet einen einfachen, gleichmäßigen Abfall, beginnend bei der Planck-Dichte, wo die Inflation sofort starten kann. Das Modell benötigt wenige freie Parameter. Insgesamt sieht es gut aus, aber leider widersprechen ihm die Daten des Planck-Satteliten. Das Plateaumodell ist dagegen viel konstruierter, braucht diffizile Feinabstimmungen. Und wie schon gesagt, es hat eine Grundschwäche, nämlich dass es einen Anfangszustand erfordert, der eigentl. erst durch die Inflation zustande kommt. Da beißt sich doch die Katze in den Schwanz.

    Ich bin ein entschiedener Vertreter des Ockhamschen Prinzips. Deswegen lehnte ich z.B. die ST ab, die auf einem ganzen Konglomerat von Hypothesen beruht. In der Geschichte der Physik war so befrachtetes Programm noch nie erfolgreich. Generell stehe ich höheren Dimensionen kritisch ggüb., genauso wie dem Multiversum. Betrachtet man sich nun Urknall mit Plateau-Inflation gegen Urprall, dann stecken doch wohl im Urknall mehr Annahmen drin. So wie ich es verstehe, sind es beim Urprall nur die vorherige Kontraktion und der Rückprall vor der Quanten-Phase. Dabei gibt es gemäß den Autoren für den Rückprall ein detailliertes Modell, ist also auch keine reine Hypothese. Andererseits wird keine Inflation mit ihren Komplexitäten und Variabilitäten gebraucht. Das Inflationsmodell ist keine präzise Theorie, sondern ein Rahmen, in dem man alles Mögliche reinstecken kann. Damit ist es kaum falsifizierbar. Falsifizerbarkeit ist für mich auch ein unabingbares Kriterium.

    Wer sagt denn, dass es das Inflatonfeld überhaupt gibt? Ursprünglich, bei Guth, waren es mehrere Higgsfelder, recht gut begründet mit der GUT. Die ist den Bach runter gegangen und dann hat man ad hoc das Inflatonfeld erfunden, quasi “aus dem Ärmel gezogen”. Was ist es denn?

  95. #95 Alderamin
    30. Mai 2017

    Steinhardt vertritt das Ekpyoritsche Universum, das ich weiter oben schon mal erwähnt habe. Es sagt also niemand (jedenfalls ich nicht), dass es keine Alternativen zur Inflationstheorie gibt. Das Schöne an der Inflation ist aber, dass sie erklärt, wo die ganze Materie herkommt (Nullenergie-Universum), wie die Feinabstimmung der Naturkonstanten zustande kommt (Anthropisches Prinzip, bei der ewigen Inflation) und natürlich das Horizontproblem, das Flachheitsproblem und das Monopolproblem löst (soweit ich es verstehe, löst das ekpyrotische Universum nur das Horizontproblem). Deswegen scheint mir das Inflationsmodell einfacher und erklärungsmächtiger, aber das ist am Ende Glaubenssache.

    Solange nicht der Nachweis der jeweils spezifischen Polarisationsmuster für die jeweiligen Modelle in der Hintergrundstrahlung gelungen ist, ist es müßig, zu spekulieren, wer Recht hat. Wir sind eh alle hier nur Laien und sollten abwarten, worauf sich die Experten einigen. Und nicht nur Steinhardt hat einen Ruf zu verlieren, Guth und Linde ebenfalls (Steinhardt war übrigens auch an der Erstellung der Inflationstheorie beteiligt).

    Über die verschiedenen Varianten der Inflationstheorie kann man übrigens im englischen Wikipedia-Artikel nachlesen. Den werde ich mir auch noch einmal zu Gemüte führen. Ist schon eine Weile her, dass ich Guths Buch gelesen habe.

  96. […] Astrodicticum Simplex: Der kalte Fleck im Kosmos und der “Beweis” für die Existenz des Multiversums […]

  97. #97 Mike
    31. Mai 2017

    Also, wenn die Chance auf ein Universum mit kaltem Fleck 2% beträgt, wäre die Wahrscheinlichkeit, dass bei 50 entstandenen Universen *mindestens* eines mit kaltem Fleck auftritt meiner Meinung nach
    1-(0.98^50), was etwa 63,6 % entspräche und somit weit von sicher entfernt ist.

  98. #98 Physik-Fan
    31. Mai 2017

    @Alderamin #95

    Das Schöne an der Inflation ist aber, dass sie erklärt, wo die ganze Materie herkommt (Nullenergie-Universum), wie die Feinabstimmung der Naturkonstanten zustande kommt (Anthropisches Prinzip, bei der ewigen Inflation) und natürlich das Horizontproblem, das Flachheitsproblem und das Monopolproblem löst (soweit ich es verstehe, löst das ekpyrotische Universum nur das Horizontproblem).

    Gemäß dem Artikel löst der Urprall auch das Gleichförmigkeitsproblem. Die Kontraktion führt schließlich zu einem flachen, gleichförmigen Universum. Wichtig dabei ist aber, dass der Übergang zur Expansion erfolgt, bevor Quantenfluktuationen wieder Dichteunterschiede reinbringen. Für diesen Übergang scheint man ein detailliertes Modell zu haben.

    Über mögliche Nachweise der Kontraktion steht nichts da, auch nicht wie die Entwicklung des Universums vorher gewesen sein könnte. Vielleicht sind diese Fragen prinzipiell unbeantwortbar, weil durch Kontraktion plus Übergang alle Informationen über das frühere Universum vernichtet werden.

    Es ist zweifelsohne eine der Stärken des Inflationsmodells, für die Feinabstimmung der Naturkonstanten eine natürliche Erklärung zu liefern. Fast zwangslos führt es zu einem Multiversum. Bei den Universumsblasen sind mir aber gewisse Zweifel gekommen. Du hast in #36 was über die Zeitpfeile gesagt, was ich nicht ganz verstehe. Der Punkt wo die Blase losging ist doch nicht das Gleiche wie ihr Außenrand danach. Dieser entspricht einem fortgeschrittenen Entwicklungsstand, also einem späteren Zeitpunkt. Klarerweise ergibt sich dann die Frage, was ist, wenn man an den Außenrand gelangt? Scheint eine sinnlose Situation zu sein. Ein Universum mit Rand macht nicht viel Sinn. Ein räumliches 3-D-Universum, das z.B. die Oberfläche einer 4-D-Kugel ist, in Ordnung. Es ist zwar auch volumenmäßig begrenzt, aber die Bewohner kommen nie an eine Grenze. Aber ein 3-D-Universum mit einer 3-D-Außengrenze ist offensichtlich Käse. Hat man das mit dem Zeitpfeil vielleicht deswegen eingeführt, um genau die Grenzsituation zu vermeiden? Durch t=0 ist der Rand unerreichbar, wäre ja eine Zeitreise in die Vergangenheit.

    Betrachten wir nun mal unser Universum, wäre ja auch so eine Blase. Wenn ich heute die Hintergrundstrahlung messe, dann ist das natürlich ein Blick in die Vergangenheit, auf die letzte streuende Oberfläche 380.000 Jahre nach dem Urknall. Aber man muss doch annehmen, dass auch am Ort der Messung die Entwicklung weitergegangen ist, dass also bei einer Momentaufnahme des Universums, nirgendwo die Hintergrundstrahlung gerade frei wird, ihre Reste “wabern” nur noch im All herum. Das gleiche muss man für den Urknall annehmen. Wo ist da ein umgekehrter Zeitpfeil? Der wäre sogar absurd, denn der Urknall war ja nicht irgendwo da “draußen”, sondern er war auch hier, in jedem Punkt unserer Umgebung.

  99. #99 Alderamin
    31. Mai 2017

    @Physik-Fan

    Du hast in #36 was über die Zeitpfeile gesagt, was ich nicht ganz verstehe. Der Punkt wo die Blase losging ist doch nicht das Gleiche wie ihr Außenrand danach. Dieser entspricht einem fortgeschrittenen Entwicklungsstand, also einem späteren Zeitpunkt.

    War nur knapp erklärt und damit schwer verständlich. Ausführlich ist es hier erklärt. Aus der Aussensicht der Blase fängt diese in einem Punkt an und wächst. Aus der Innensicht mit geändertem Zeitpfeil ist die gesamte Außenhaut der Blase über ihre gesamte Lebensdauer (aus der Außensicht) ein einzelner Zeitpunkt (nämlich der Urknall) in einem unendlich großen Raum.

    Stell’ Dir als senkrechte Achse die externe Zeitachse und als waagerechte Ebene die drei externen Raumdimensionen des inflationären Raums vor, reduziert auf 2. Dann ist die wachsende Blase ein auf der Spitze stehender Kegel. Jede Scheibe des Kegels ist die Blase zu einer anderen externen Zeit. Die Hülle des Kegels ist unendlich groß, weil er nach oben offen ist. Aus der Innensicht der Blase ist die Hülle ein unendlich großer Raum zur Zeit des Urknalls und der Zeitpfeil zeigt schräg nach oben und nach innen. Das Universum fängt hier also nicht als Punkt an, sondern ist bei seiner Entstehung schon unendlich groß – dennoch wächst in ihm jedes Volumen gemäß der Hubble-Expansion. Sprich, die Zeitrichtung des umgebenden inflationären Raums wird zur Raumrichtung in der Blase. Details siehe Link.

    Klarerweise ergibt sich dann die Frage, was ist, wenn man an den Außenrand gelangt?

    Da man nicht in die Vergangenheit reisen kann, kann man den Außenrand nicht erreichen. Und wenn mans könnte, landete man im Feuerball des Urknalls, der alles zerlegt. Das Blasen-Universum hat keinen räumlichen Rand, nur einen zeitlichen Anfang.

    Hat man das mit dem Zeitpfeil vielleicht deswegen eingeführt, um genau die Grenzsituation zu vermeiden?

    Nein, sondern der Zeitpfeil zeigt einfach in Richtung steigender Entropie. Die Entropie am Rand der Blase ist immer und zu allen (externen) Zeiten gleich groß und minimal. Man muss sich dazu mal überlegen, was Zeit eigentlich ist. Eine gängige Erklärung für die Vorzugsrichtung der Zeit ist, das der Zeitpfeil in Richtung steigender Entropie zeigt. Deswegen kann man leicht eine Vase auf dem Boden zertrümmern, aber sie setzt sich nicht so leicht wieder zusammen, obwohl man eigentlich nur alle Bewegungen und Kräfte der beteiligten Partikel umkehren müsste – die physikalischen Gesetze der Bewegung sind bzgl. der Zeitrichtung symmetrisch. Auch unsere Wahrnehmung basiert auf Prozessen des Gehirns, die Energie verbrauchen und damit die Entropie erhöhen. Das Vergehen von Zeit könnte genau so gut einen Illusion des Gehirns sein.

    Betrachten wir nun mal unser Universum, wäre ja auch so eine Blase. Wenn ich heute die Hintergrundstrahlung messe, dann ist das natürlich ein Blick in die Vergangenheit, auf die letzte streuende Oberfläche 380.000 Jahre nach dem Urknall. Aber man muss doch annehmen, dass auch am Ort der Messung die Entwicklung weitergegangen ist, dass also bei einer Momentaufnahme des Universums, nirgendwo die Hintergrundstrahlung gerade frei wird, ihre Reste “wabern” nur noch im All herum. Das gleiche muss man für den Urknall annehmen. Wo ist da ein umgekehrter Zeitpfeil? Der wäre sogar absurd, denn der Urknall war ja nicht irgendwo da “draußen”, sondern er war auch hier, in jedem Punkt unserer Umgebung.

    Die Frage verstehe ich nicht. Wir befinden uns im Inneren der Blase und unser Blick in die Ferne geht zurück Richtung Rand, wo er an der Hintergrundstrahlung seine Grenze findet. Den Ort in der Hintergrundstrahlung, wo unsere Ecke hier entstand, sehen wir nicht, das Licht hat sich längst davon gemacht, wir sehen nur solche Orte, die so weit weg sind, dass das Licht von dort gerade hier vorbei kommt. In der Blase gelten natürlich Lichtkegel wie in jedem Universumsmodell. Wo ist jetzt das Problem?

    Und zum Schluss: Auch das ist nur eine von vielen Erklärungsvarianten, die erklärt, wie ein Universum unendlich groß begonnen haben kann. Muss es aber nicht. Wir sind nur ziemlich sicher, dass das beobachtbare Universum nicht alles ist, sondern nur ein Horizont innerhalb eines sehr viel größeren Universums.

  100. #100 Physik-Fan
    1. Juni 2017

    @Alderamin #99

    Ich hatte doch die richtige Vermutung, dass es mit der Plausibilität der Universumsblasen zu tun hat. Diese hatte ich auch hingenommen, ohne groß darüber nachzudenken, aber dann fiel mir auf, dass ein Universum mit Außengrenze Quatsch ist, macht keinen Sinn. Anderen kam das natürlich auch. Aguirre ist wohl einer davon und hat was zusammengebastelt, denn so sieht es auch aus, wie etwas künstlich Konstruiertes. Wer sagt denn, dass da irgendwas dran ist? Ich habe auch Verdacht, dass es nicht mal konsistent ist. Aber zugegeben, die Konstruktion und ihre Implikationen überblicke ich noch nicht. Aber was ich sehe, das sind Widersprüche zu unserem Universum. Das wäre ja nach der Theorie so eine Blase.

    Das Problem ist für die Inflationstheorie, dass sie letztendlich an den Blasen hängt. Die Inflation ist nicht zu stoppen, was auch Steinhardt im Artikel sagt. Also können Universen mit normalem Vakuum nur eingelagert sein und damit ist Crux da.

    Da man nicht in die Vergangenheit reisen kann, kann man den Außenrand nicht erreichen.

    Genau das war ja mein Schluss. Betrachten wir unser Universum. Den Urknall als ewigen Zeitpunkt 0 zu betrachten, ist doch absurd. Er ist auch nicht an einem “Außenrand”, sondern er ist überall in dem Sinne, dass er an jedem Ort des Universums stattgefunden hat. Man veranschaulicht die Expansion typischerweise mit dem Ballonbild. Daran sieht man es sofort.

    Aber nehmen wir an, der Urknall wäre “irgendwo” und immer am Zeitanfang. Es muss ja dann irgendwie zu unserer Zeit t=13,8 Mia. Jahre kommen, d.h. es ergäbe sich eine irgendwie geartete zeitliche Zunahme längs des Weges vom Urknall zu uns. Daraus folgt, dass an Orten näher zum Urknall die Zeit langsamer läuft, wie bei uns. Was könnte das für Auswirkungen haben? Nehmen wir die Atomspektren, sie wären zu niedrigeren Frequenzen verschoben, bei ausreichender Entfernung locker unterhalb des Lichtspektrums. Dazu käme noch die Verschiebung durch die Fluchtbewegung. Gibt es irgendwelche Befunde für diese Gesamtverschiebung? Ich bezweifele das.

    Eine gängige Erklärung für die Vorzugsrichtung der Zeit ist, dass der Zeitpfeil in Richtung steigender Entropie zeigt.

    Entropie ist ein Maß für die Wahrscheinlichkeit eines Zustandes. Dass Materie in Form einer Tasse vorliegt, ist eben weniger wahrscheinlich, als in Bruchstücken davon. Das ist das ganze Geheimnis an der Entropie. So gesehen eigentl. trivial und auch nicht fundamental.

    Der Zeitpfeil ist für mich mit der Kausalität verknüpft, dass die Zeit entlang von Ursache-Wirkungs-Ketten verläuft. Natürlich kann so eine Kette prinzipiell auch umgekehrt verlaufen, aber es kommt auf den tatsächlichen Ablauf an. Für das Universum ist es die Entwicklung vom Urknall bis heute. Anzunehmen, dass es in Wirklichkeit umgekehrt ist, dass der heutige Zustand sich zum Urknall entwickelt, widerspricht der Erfahrung. Bspwse. wären in unserer Erinnerung Dinge, die es noch gar nicht gibt, anderseits wäre nichts Vergangenes drin. Smolin vertritt übrigens diese Sicht auf die Zeit, s. “Im Universum der Zeit”. Er plädiert dafür, die Zeit so aufzufassen, wie wir sie erleben, wie es unserer Intuition entspricht.

  101. #101 Steffmann
    2. Juni 2017

    @Physik-Fan:

    Den Urknall als ewigen Zeitpunkt 0 zu betrachten, ist doch absurd

    Ja, das ist es in der Tat, da es da keine Zeit gab.

    Entropie ist ein Maß für die Wahrscheinlichkeit eines Zustandes

    Also, von allen gängigen Definition in den Fakultäten zur Entropie, habe ich das noch nicht gehört. Aber gut. Hier ist gemeint: Tendenz in der unbelebten Welt zu immer größerer Unordnung.

  102. #102 Frantischek
    2. Juni 2017

    Der Zeitpfeil ist für mich mit der Kausalität verknüpft, dass die Zeit entlang von Ursache-Wirkungs-Ketten verläuft.

    Das würde z.B. heißen dass es keine Hintergrundstrahlung geben kann, weil damals keine Zeit vergangen ist.
    Die ist ja durch die Rekombination (doofes Wort, die waren ja vorher frei, sollte also nur Kombination heißen) der Elektronen entstanden.

    Wenn du da Kausalität findest, also sagen kannst wann welches Elektron sich an welches Proton bindet, dann auf nach Stockholm!

  103. #103 Alderamin
    2. Juni 2017

    @Physik-Fan

    dann fiel mir auf, dass ein Universum mit Außengrenze Quatsch ist, macht keinen Sinn.

    Es hat ja keine Außengrenze, nur eine Zeitgrenze. Im übrigen ist das, wie gesagt, nicht DIE Inflationstheorie, sondern eine vieler Varianten. Genau so gut könnte man auch annehmen, dass eine Blase ein 4D-Torus oder eine Hyperkugel ist, deren Oberfläche unser 3D-Raum ist. Auch das sind gängig diskutierte Hypothesen. Hast Du vermutlich schon von gehört.

    Aguirre ist wohl einer davon und hat was zusammengebastelt, denn so sieht es auch aus, wie etwas künstlich Konstruiertes.

    Die Grundidee war in Brian Greenes “Hidden Reality” erklärt, aber als Link im Netz habe ich Aguirres Seite gefunden. Ob die Idee direkt von ihm kommt, weiß ich nicht. “Sieht aus wie künstlich konstruiert” ist ein komisches Argument. Modelle sind immer konstruiert, und zwar so, dass sie eine Beobachtung erklären und Vorhersagen machen können. Die Konstruktion hier erlaubt die Entstehung eines unendlich großen Raums zur Zeit des Urknalls, das ist konsistent mit der Beobachtung, dass der Raum im Rahmen der Messgenauigkeit flach ist. Er könnte aber auch einfach nur sehr groß mit endlichem Volumen sein (4D-Torus, Hyperkugel o.ä.).

    Es muss ja dann irgendwie zu unserer Zeit t=13,8 Mia. Jahre kommen, d.h. es ergäbe sich eine irgendwie geartete zeitliche Zunahme längs des Weges vom Urknall zu uns. Daraus folgt, dass an Orten näher zum Urknall die Zeit langsamer läuft, wie bei uns.

    Da kann ich nicht folgen. Die Zeit läuft schräg nach innen oben im Raumzeit-Kegel. Über die Geschwindigkeit ist gar nichts gesagt worden.

    Nehmen wir die Atomspektren, sie wären zu niedrigeren Frequenzen verschoben, bei ausreichender Entfernung locker unterhalb des Lichtspektrums. […] Gibt es irgendwelche Befunde für diese Gesamtverschiebung?

    Es gibt Messungen dazu, dass sich die Lichtgeschwindigkeit über große Zeiten des Weltalters nicht verändert hat (Messungen der Feinstrukturkonstanten).

    Entropie ist ein Maß für die Wahrscheinlichkeit eines Zustandes. Dass Materie in Form einer Tasse vorliegt, ist eben weniger wahrscheinlich, als in Bruchstücken davon. Das ist das ganze Geheimnis an der Entropie. So gesehen eigentl. trivial und auch nicht fundamental.

    Und genau das wird als Ursache dafür angesehen, dass es einen Zeitpfeil mit einer Vorzugsrichtung gibt, denn es gibt mehr Möglichkeiten, in die eine Richtung zu gehen, als zurück. Gas aus einer Druckflasche mit geöffnetem Hahn kann sich auf mehr Arten im Raum verteilen, als es das in der Flasche kann, also verteilt es sich eher, als dass es sich komprimiert. Auch unsere Wahrnehmung funktioniert, wie gesagt, in dieser Richtung. Zeit könnte eine Illusion sein. Martin hat schon mal einen Artikel darüber geschrieben.

    Der Zeitpfeil ist für mich mit der Kausalität verknüpft, dass die Zeit entlang von Ursache-Wirkungs-Ketten verläuft.

    Und diese Ketten funktionieren eben prima in beiden Richtungen. Wenn man einen Film über einen elementaren physikalischen Vorgang rückwärts laufen ließe, dann stellte er wieder einen korrekten physikalischen Vorgang dar. Nur in der Masse werden solche Prozess durch die Statistik unwahrscheinlich, wenn sie die Entropie senken würden.

    Natürlich kann so eine Kette prinzipiell auch umgekehrt verlaufen, aber es kommt auf den tatsächlichen Ablauf an.

    Der, wie gesagt, eine Illusion sein könnte. Man stelle sich vor, in der Zeitdimension sei alles starr und fest vorgegeben, wie im Blockuniversum (siehe Martins Artikel). Dann entsteht Zeit erst dadurch, dass sich die Wahrnehmung entlang der Zeitachse entwickelt. Als ob man in einem Zug an einer Landschaft vorbeifährt. Kausalität ist dann einfach der Zusammenhang von Zuständen entlang der Zeitachse. Und der besteht in beiden Richtungen. Aber die Wahrnehmung läuft in Richtung steigender Entropie, also sehen wir die Zeit in dieser Richtung fortschreiten.

    Du kannst das alles für Unsinn halten, aber das sind Ansichten, die durchaus intelligente Physiker und Philosophen vertreten. Natürlich gibt’s auch andere Meinungen. Das Modell funktioniert, ist aber praktisch kaum experimentell zu verifizieren. Ich glaube, im Bereich der Quantenradierer versucht man entsprechende Experimente zu entwickeln. Lies mal Greenes Hidden Reality, da steht alles drin. Eine Rezension dazu gibt’s bei Florian.

  104. #104 Physik-Fan
    2. Juni 2017

    @Steffmann #101

    Also, von allen gängigen Definition in den Fakultäten zur Entropie, habe ich das noch nicht gehört. Aber gut. Hier ist gemeint: Tendenz in der unbelebten Welt zu immer größerer Unordnung.

    Schon mal was von der statistischen Mechanik von Boltzmann gehört? Siehe den Wikipedia-Artikel https://de.wikipedia.org/wiki/Ludwig_Boltzmann. Dort heißt es “… so können wir diejenige Größe, welche man gewöhnlich als die Entropie zu bezeichnen pflegt, mit der Wahrscheinlichkeit des betreffenden Zustandes identifizieren.“ In exakter Form drückt das seine berühmte Formel aus:

    S = k * log W

  105. #105 Physik-Fan
    2. Juni 2017

    @Frantischek #102

    Die ist ja durch die Rekombination (doofes Wort, die waren ja vorher frei, sollte also nur Kombination heißen) der Elektronen entstanden.

    Wenn du da Kausalität findest, also sagen kannst wann welches Elektron sich an welches Proton bindet, dann auf nach Stockholm

    Du vermengst Elementarereignisse, wo die Quantenphysik gilt, mit Makrophysik. Dass Atomkerne und Elektronen überhaupt kombinieren, liegt doch wohl an der elektromagn. Wechselwirkung zwischen ihnen. Diese gegenseitig anziehende Kraft ist natürlich die Ursache. Angenommen Atomkerne wären elektrisch neutral, würden sie dann auch mit Elektronen kombinieren?

    Für das Einzelereignis Elektron A wird von Kern B eingefangen, gibt es nur eine Wahrscheinlichkeit, es ist also nicht streng bestimmt, aber die Wahrscheinlichkeit sagt ja, dass man für eine Anzahl von Annäherungen im statistische Mittel eine bestimmte Zahl von Einfängen hat. Gemäß dem Gesetz der großen Zahl wird es sich bei steigender Anzahl immer mehr der Wahrscheinlichkeit näher. Und damals, nach 380.000 Jahren, gab es seeeeeeehr viele freie Kerne und Elektronen.

    So gesehen kann man Kausalität als emergentes Phänomen betrachten. Dass Kausalität in der Makrophysik prinzipiell gilt, daran besteht kein Zweifel. Schalte mal den Föhn ein und er wird losgehen (wenn technisch alles in Ordnung ist), obwohl für jedes einzelne Elektron im Leiter nur die Regeln der Quantenphysik gelten.

    Eine große Frage war ja immer, wie genau geht die Quantenphysik in die klassische Physik
    über? Diese Diskussion anzufangen, würde jetzt zu weit führen. Ich will nur mal auf den Artikel “Schrödingers Katze auf dem Prüfstand”, Spektrum 10.2012 verweisen. Mit der Dekohärenz der Wellenfunktion ist man zumindest etwas weitergekommen, aber die Frage bleibt weiterhin spannend.

  106. #106 Steffmann
    2. Juni 2017

    @Physik-Fan:

    Schon mal was von der statistischen Mechanik von Boltzmann gehört?

    Du weisst nicht wirklich, wovon Du redest oder ?

  107. #107 Steffmann
    2. Juni 2017

    Nix für ungut. Boltzmann war ein Stochastiker. Das hat mit dem Thema hier nichts zu tun. Auch wenn Du mit dem Hinweis, dass er Entropie erwähnte, Recht hast.

  108. #108 Steffmann
    2. Juni 2017

    Letzten Endes geht es hier nur um die Frage, ob die Inflation plausibel genug ist, um als zukünftige Theorie herhalten zu können. Das ist sie meiner Meinung nach, da es kein plausibleres Erklärungsmodell gibt. Die wirre und ziemlich durchgemischte (oft nicht nachvollziehbare) Argumentation von Physik-Fan ändert daran nichts. In einer Umgebung, in der Universen unendlich ausgespuckt werden, ist auch die zufällige Verteilung der Naturkonstanten nichts mehr besonders.

  109. #109 Physik-Fan
    3. Juni 2017

    Steffmann #107, #108

    Boltzmann war ein Stochastiker. Das hat mit dem Thema hier nichts zu tun. Auch wenn Du mit dem Hinweis, dass er Entropie erwähnte, Recht hast.

    Er war nicht einfach ein Stochastiker und er hat die Entropie nicht nur mal so erwähnt, sondern ihr eine neue Sichtweise, eine neue Definition gegeben. Er hat den zweiten Hauptsatz der Thermodynamik aus statistischen Betrachtungen abgeleitet.

    Letzten Endes geht es hier nur um die Frage, ob die Inflation plausibel genug ist, um als zukünftige Theorie herhalten zu können. Das ist sie meiner Meinung nach, da es kein plausibleres Erklärungsmodell gibt. Die wirre und ziemlich durchgemischte (oft nicht nachvollziehbare) Argumentation von Physik-Fan ändert daran nichts. In einer Umgebung, in der Universen unendlich ausgespuckt werden, ist auch die zufällige Verteilung der Naturkonstanten nichts mehr besonders.

    Ich gebe nur wieder, was Steinhardt und Mitstreiter sagen. Lese den referenzierten Artikel in der Spektrum (hast Du vermutl. noch nicht) und wende Dich zwecks Einwänden direkt an die Autoren.

    Was Deinen letzten Satz angeht, gebe ich Dir recht. Aber was schön und wünschenswert ist und was stimmt, ist leider nicht immer das Gleiche. Ein Multiversum kann es auch auf anderer Basis als der Inflation geben, z.B. die Branenwelt von Randall/Sundrum.

  110. #110 Physik-Fan
    3. Juni 2017

    @Alderamin #103

    Genau so gut könnte man auch annehmen, dass eine Blase ein 4D-Torus oder eine Hyperkugel ist, deren Oberfläche unser 3D-Raum ist. Auch das sind gängig diskutierte Hypothesen.

    So könnte ich es mir auch vorstellen. Mal ein Versuch der Veranschaulichung und zwar übersetzt um eine Dimension tiefer. Die globale Topologie könnte z.B. der Oberfläche einer Kugel entsprechen, die sich exponentiell ausdehnt. Taschenuniversen wären als kugelförmige Ausbeulungen denkbar. Genauer gesagt, ein Taschenuniversum wäre die Oberfläche der Ausbeulung und würde somit, trotz räumlicher Begrenzung, die Bedingung der Unbegrenztheit für ihre Bewohner erfüllen.

    “Sieht aus wie künstlich konstruiert” ist ein komisches Argument.

    Naja, mir kommt’s tricky vor, aber vielleicht verstehe ich es nur nicht richtig. Wichtiger ist die Frage, ob es ein rein hypothetisches Konstrukt ist oder ob es zwangsläufig so ist. Das Zweite wäre gut, aber vermutl. wird’s das Erste sein. Dann packt man halt weitere Hypothesen drauf. Das war auch ein Kritikpunkt von Steinhardt, die Anzahl der Annahmen, der freien Parameter.

    Da kann ich nicht folgen. Die Zeit läuft schräg nach innen oben im Raumzeit-Kegel. Über die Geschwindigkeit ist gar nichts gesagt worden.

    Ich hab’s wohl missverständlich ausgedrückt. Mit Geschwindigkeit hat es nichts zu tun. Nehmen wir eine Linie vom Außenrand zu uns und betrachten den Stand von Uhren längs der Linie. Am Außenrand gilt t=0, bei uns t=13,8 Mia. Jahre, zwischendrin müssen Uhren eine Zeit dazwischen anzeigen. Dort ist also seit dem Urknall weniger Zeit vergangen oder anders ausgedrückt, dort läuft die Zeit langsamer wie bei uns. Mache ich da einen Denkfehler?

    Aber das ist nur ein Detail, generell ist m.E. dieses Modell wenig plausibel, plausibler ist es gemäß dem Ballonbild, dass bei einer Momentaufnahme überall im Universum der Zeitabstand zum Urknall gleich ist und auch der Uhrengang gleich ist (lokale relativistische Effekte mal ausgenommen).

    Man stelle sich vor, in der Zeitdimension sei alles starr und fest vorgegeben, wie im Blockuniversum (siehe Martins Artikel). Dann entsteht Zeit erst dadurch, dass sich die Wahrnehmung entlang der Zeitachse entwickelt. Als ob man in einem Zug an einer Landschaft vorbeifährt. Kausalität ist dann einfach der Zusammenhang von Zuständen entlang der Zeitachse. Und der besteht in beiden Richtungen. Aber die Wahrnehmung läuft in Richtung steigender Entropie, also sehen wir die Zeit in dieser Richtung fortschreiten.

    Wir nehmen das wahr, was sich ergibt (im Idealfall jedenfalls; aber man könnte auch objektiv, maschinell aufzeichnen) und das ist nun mal öfters der wahrscheinlichere Fall, als der weniger wahrscheinliche. Dass die Tasse sich wieder zusammenfügt, erlebt man deswegen nicht, weil die Wahrscheinlichkeit dafür so extrem gering ist, aber unmöglich ist es nicht, es könnte passieren und dann würde man es so registrieren. Warum sollte es nicht so sein? Gut, der Fall wird wohl nie eintreten, aber man kann das auch mit Anordnungen durchspielen (z.B. mit Würfeln) wo die Wahrscheinlichkeiten nicht so extrem auseinander liegen und das Eintreten von weniger wahrscheinlichen Fällen im Experiment realistisch ist. Dann wird man Fälle beobachten, die der Entropiezunahme entsprechen und andere nicht. Der Wahrnehmung ist die Entropie eben wurscht. Entropie ist sowieso keine eigene Qualität, sie ist nur ein Maß für die Wahrscheinlichkeit. Die Entropie nimmt deswegen zu, weil eben das Wahrscheinlicherere öfters eintritt. Das gilt natürlich nur im statistischen Mittel, aber bei steigender Anzahl der Fälle – und in der Realität hat man es oft mit seeeeeher vielen Fällen zu tun – wird es sich mit immer besserer Näherung so ergeben.

    Nun zur eventuellen Illusion des Ablaufs einer Ursache-Wirkungs-Kette. Man lege ein Protokoll der sich ergebenden Zustände an: A, B, C, D, E …. Bei umgekehrten Verlauf sieht das Protokoll so aus: …. E, D, C, B, A. Die Protokolle sind verschieden. Unser Gedächtnis ist so ein Protokoll. Wenn im Gedächtnis der erste Verlauf ist, aber in Wirklichkeit der Ablauf umgekehrt wäre, dann wären die Gedächtnisinhalte äußerst seltsam. Man würde sich an Dinge erinnern, die noch gar nicht eingetreten sind und andererseits an nichts Vergangenes. Sinnvolles Handeln wäre so nicht möglich.

    Nochmals meine Empfehlung von Smolins “Im Universum der Zeit”, wo es um eine Revision des Zeitbegriffes der Physik geht und zwar genau im dargelegten Sinne, dass der erlebte Zeitverlauf nicht nur eine Illusion ist, sondern Realität.

  111. #111 Alderamin
    3. Juni 2017

    @Physik-Fan

    Dort ist also seit dem Urknall weniger Zeit vergangen oder anders ausgedrückt, dort läuft die Zeit langsamer wie bei uns. Mache ich da einen Denkfehler?

    Ja, das ist einfach die Vergangenheit. Wenn der Zeitpfeil zu uns führt, ist es unsere Vergangenheit, sonst die eines anderen Orts.Die Blase fängt als Punkt an und dehnt sich dann aus (aus der Außensicht). Irgendwann (Außenzeit) kommt der Moment, wo der für unseren Ort relevante Urknall stattfindet und von da an läuft unser Zeitpfeil nach der Innensicht los und geht ins Innere der Blase hinein. Die Zeit vergeht 13,8 Milliarden Jahre lang und dann sind wir am heutigen Ende des Pfeils. Aus unserer Sicht liegen gleiche Zeiten auf Kugelschalen innerhalb der Blase. Jede solche Schale ist allerdings tatsächlich ein 3D-Raum.

    plausibler ist es gemäß dem Ballonbild, dass bei einer Momentaufnahme überall im Universum der Zeitabstand zum Urknall gleich ist und auch der Uhrengang gleich ist (lokale relativistische Effekte mal ausgenommen).

    Kann das Ballonbild denn einen unendlichen Raum zur Zeit des Urknalls abbilden? Ich denke nicht. Das ist aber eine valide Annahme, die man nach den derzeitigen Beobachtungen nicht ausschließen kann.

    Zum Zeitbegriff nochmal zwei Links. Aus einem geordneten Zustand sollte eigentlich in beliebiger Zeitrichtung nur ein ungeordneterer hervorgehen. Das ist nur dann anders, wenn es einen Anfangszustand mit größtmöglicher Ordnung gab, aus dem in jeder Zeitrichtung nur ungeordnetere Zustände hervorgehen können. Dies definiert einen Zeitpfeil vom Anfangszustand weg. Der Urknall war ein solcher Anfangszustand. Von ihm weg weist also ein Zeitpfeil, der für uns dann als Nachkömmlinge relevant ist. So ungefähr ging das Argument in Brian Greenes Buch, und so steht es sinngemäß auch hier und hier. Und das gilt dann eben auch für die Universumsblase, von außen nach innen.

  112. #112 Frantischek
    4. Juni 2017

    @Physikfan:
    Dass Kausalität gilt (oder es kann) sobald man genug einzelne Quantenereignisse betrachtet stimmt.
    Ob man das 1:1 so umlegen kann wie ich oder du es tun, da bin ich mir nicht sicher. Deswegen streit ich besser nicht drum…

    Trotzdem bin ich mir ziemlich sicher dass man den Zeitpfeil nicht an der Kausalität festmachen kann.

    1. Vergeht auch für einzelne Teilchen Zeit.
    2. Vergeht auch für Systeme die nicht kausal zusammenhängen Zeit (siehe z.B. Horizontproblem).
    3. Vergeht auch bei eindeutig nicht determinierten Quantenereignissen Zeit.

    Das soll dann vielleicht noch einmal wer beantworten der davon mehr Ahnung hat als ich.

    Unabhängig davon sollte dir das hier gefallen:
    https://scienceblogs.de/hier-wohnen-drachen/2012/08/19/roger-penroses-zyklisches-universum/

  113. #113 Physik-Fan
    4. Juni 2017

    @Frantischek #112

    1. Vergeht auch für einzelne Teilchen Zeit.
    2. Vergeht auch für Systeme die nicht kausal zusammenhängen Zeit (siehe z.B. Horizontproblem).
    3. Vergeht auch bei eindeutig nicht determinierten Quantenereignissen Zeit.

    zu 1.) Warum sollte für ein elementares Teilchen (wie das Elektron) Zeit relevant sein? Es hat keine innere Struktur und alle seine Prozesse sind zufällig. Jedes teilchenphysikalische Ereignis kann auch umgekehrt ablaufen. Allerdings, ganz streng gilt es nicht. Es gibt die Verletzung der CP-Symmetrie, welche zuerst bei den K0-Mesonen entdeckt wurde. Da nach heutiger Kenntnis die CPT-Symmetrie streng gilt, muss auch die T-Symmetrie, also die Zeitumkehrinvarianz, verletzt sein. Ob das nur was Exotisches ist oder fundamentale Bedeutung hat, weiß vermutl. niemand.

    zu 2.) Es ergibt sich die Frage, ob die beiden Systeme unterschiedliche Zeitpfeile haben könnten, z.B. in zwei entfernten Galaxien. Sehr plausibel ist das nicht. Letztendlich kommt man dazu, dass wohl auch die Ursachen-Wirkungs-Kette nur ein Indikator ist, aber nicht der Zeitablauf selbst. Smolin scheint doch recht zu haben, mit seiner absoluten Zeit (sein Buch lesen!).

    zu 3.) s. Antwort auf 1.

    Wenn die Ursachen-Wirkungs-Kette keine zeitliche Relevanz hat, dann muss mir einer mal erklären, warum es in unserem Gedächtnis eine bestimmte Vergangenheitsrichtung gibt und zwar nicht nur individuell, sondern für alle Menschen einheitlich. Oder gibt’s Leute, in deren Erinnerung die Zukunft ist oder sogar beides? In den Kreisen von Esoterikern mag es ja so was geben …

    Außerdem, was soll an Entropie fundamental sein? Die Entropiezunahme als eine reine Konsequenz von Wahrscheinlichkeiten, ist komplett trivial. Die universelle Zeitreversibilität von Prozessen macht es möglich, dass sich die Entropie auch mal verringert. Soll dann dort die Zeit plötzlich umgekehrt laufen? Das anzunehmen, ist doch grotesk.

    Der Heckmeck um die Entropie ist mir eh unklar. Das wird z.B. gerätselt um die Rolle der Gravitation, weil sie ja strukturbildend ist. Vom Standpunkt der Wahrscheinlichkeit her, ist das null Problem. Für ein gravitatives System ist der Zustand der minimalen potentiellen Gesamtenergie am wahrscheinlichsten und den wird es, für sich alleine gelassen, irgendwann annehmen.

  114. #114 Frantischek
    5. Juni 2017

    Zu 1:
    Siehe z.B. den von mir verlinkten Artikel von Martin (der ein Buch von Roger Penrose rezensiert):

    ein einziges massives Teilchen würde quasi als “Uhr” wirken können (weil die Ruhemasse des Teilchens einer Energie und die wiederum einer Frequenz entspricht)

    Feynman Diagramme haben ja auch immer eine Zeitachse.

    Es hat keine innere Struktur und alle seine Prozesse sind zufällig.

    Ob ein Elektron wirklich keine innere Struktur hat oder uns diese einfach noch nicht zugänglich ist, ist meines Wissens nicht geklärt.
    Und z.B. die Frequenz ist sicher nicht zufällig, und beschreibt eine Schwingung = Prozess.

    Zu 2:
    Warum ergibt sich die Frage?
    Es geht darum ob der Zeitpfeil auf der Kausalität beruht, es gibt große Teile des Universums die kausal nicht zusammenhängen können, in diesen Teilen vergeht genau so Zeit. Punkt.

    Nebenbei wissen wir recht genau dass die Zeit, in gegenüberliegenden Teilen des Universums die weeeeiiit auseinander liegen, gleich vergeht. Unterschiede würde man in Spektren oder am Ablauf von Supernovas erkennen.

    Zu 3:
    Das enthält keine Aussage.
    Findest du jetzt Entropie doof, oder nur die Rolle die ihr zugewiesen wird?
    Besonders im letzten Teil beschreibst du nix anderes als Entropie (die Tendenz einen wahrscheinlicheren Zustand anzunehmen, welcher in den allermeisten Fällen ein ungeordneterer als der vorige ist).

  115. #115 Physik-Fan
    5. Juni 2017

    @Frantischek #112

    3. Vergeht auch bei eindeutig nicht determinierten Quantenereignissen Zeit.

    Eine Ergänzung, zur begrifflichen Präzisierung. Zufälligkeit und Determinismus sind zwei verschiedene Eigenschaften. Determinismus bedeutet Unbestimmtheit oder Unbestimmbarkeit und in der Quantenphysik ist es die prinzipielle Unbestimmtheit bestimmter Größen in ihrer Kombination. Wir reden hier natürlich über die Heisenbergsche Unbestimmtheitsrelation. Das bedeutet, dass für ein Quantenobjekt, wie ein elementares Teilchen, etwa Ort und Impuls nicht zugleich scharf bestimmt sind und zwar nicht aus messtechnischen Gründen, sondern als inhärente Eigenschaft des Objekts selbst. Dieser Indeterminismus ist eine Aussage über den Zustand und so wohl keine über den Verlauf der Zeit.

  116. #116 Frantischek
    5. Juni 2017

    Determinismus bedeutet Unbestimmtheit oder Unbestimmbarkeit …

    Nope. Genau das Gegenteil. Aber ich denke das ist ein Tippfehler…

    Dieser Indeterminismus ist eine Aussage über den Zustand und so wohl keine über den Verlauf der Zeit.

    Das eine hat doch mit dem anderen nix zu tun.
    Es ist egal WANN oder WARUM sich ein Zustand ändert, wenn er es tut muss Zeit vergangen sein (selbst wenn die Zustandsänderung instantan erfolgt).

    Wie gesagt vergeht aber auch für ein einzelnes Teilchen Zeit. Selbst wenn keine Interaktion oder Zerfall stattfindet.

    Schau dir die Formeln an die die Energie eines Teilchens beschreiben (ich kann hier keine Formeln einfügen). Da ist überall die Wellenfrequenz drin, die wiederum die Zeit enthält.

    Da gehts nur um die Energie, da muss das Teilchen noch nix tun, es muss nix mit ihm passieren, und schon gar nicht muss es mit einem anderen interagieren.

    Wo ist da die Kausalität?

  117. #117 Physik-Fan
    5. Juni 2017

    @Frantischek #114, #116

    ein einziges massives Teilchen würde quasi als “Uhr” wirken können (weil die Ruhemasse des Teilchens einer Energie und die wiederum einer Frequenz entspricht)

    Du willst also z.B. an einem Elektron die Zeit ablesen. Wie erkennt man das an ihn? Um es zu präzisieren: Am Elektron SELBST, nicht z.B. am Verlauf seiner Bahn. Die Quantenwelt ist so seltsam, dass man nicht mal von ein und demselben Teilchen auf der Bahn sprechen kann. Ein Elektron hat keine Identität im Sinne einer individuellen Historie. Deswegen kann es prinzipiell keine Uhr darstellen. Übrigens, auf der fehlenden Identität von Teilchen beruht die Einstein-Bose-Statistik und die aus dem Pauli-Prinzip folgende Ausschließungsbedingung für Teilchen mit halbzahligem Spin.

    Ob ein Elektron wirklich keine innere Struktur hat oder uns diese einfach noch nicht zugänglich ist, ist meines Wissens nicht geklärt.

    Das Elektron gilt als elementar, das ist völlig unstrittig.

    Es geht darum ob der Zeitpfeil auf der Kausalität beruht, es gibt große Teile des Universums die kausal nicht zusammenhängen können, in diesen Teilen vergeht genau so Zeit.

    Ich sage ja, dass die Kausalität ein Indikator ist, wie die Zeit verläuft, aber nicht die Zeit selbst. Der Zeitablauf ist möglichweise fundamental, nicht emergent. Smolin vertritt das jedenfalls. Lese mal das schon referenzierte Buch von ihm. Es lohnt sich!

    Findest du jetzt Entropie doof, oder nur die Rolle die ihr zugewiesen wird?

    Was soll an einem Maß für die Wahrscheinlichkeit eines Zustandes doof sein? Aber Entropie ist ungeeignet, den Zeitpfeil daran festzumachen. Nochmals das Beispiel mit der zerbrochenen Tasse, die sich wieder zusammenfügt. Die Wahrscheinlichkeit dafür ist extrem gering ist, aber es ist prinzipiell möglich und wenn es passiert, wird man es auch so registrieren. Wenn es nicht die Entropie ist, was könnte es sonst sein? Mit fällt nur der Ursache-Wirkungs-Zusammenhang ein. Dass die Abfolge von Ursache und Wirkung nicht nur eine Illusion ist, zeigt doch unser Gedächtnis. Alle Menschen erleben die gleiche Vergangenheitsrichtung in ihrer Abfolge von Ursachen und Wirkungen. Man könnte es auch in objektiver Form durch ein Protokoll machen.

    Es ist egal WANN oder WARUM sich ein Zustand ändert, wenn er es tut muss Zeit vergangen sein (selbst wenn die Zustandsänderung instantan erfolgt).

    Es geht nicht um Zustandsänderungen sondern um den Zustand. Ein Quantenobjekt ist in bestimmter Hinsicht in keinem scharf definierten Zustand.

    Wo ist da die Kausalität?

    Ein Teilchen wechselwirkt nicht in kausaler Art (z.B. Emission/Absorption eines virtuellen Photons durch ein Elektron), aber in der Makrophysik ist Kausalität ein sinnvolles Konzept. Nochmals der Föhn. Wenn man ihn gerade eingeschaltet hat und er läuft los, was ist die Ursache? Doch wohl, dass man den Einschaltknopf betätigt hat. Man könnte ja mal den Versuch machen, den Föhn nur anzuschließen und liegen zu lassen, vielleicht geht er irgendwann von alleine los …

  118. #118 Frantischek
    6. Juni 2017

    Deine Antworten sind teilweise wirr, in den allermeisten Fällen gehen sie komplett am Thema vorbei.

    Nur noch soviel, den Rest lernst du bitte an der Uni:

    In welchem Semester sagtest du bist du?

    Das Elektron gilt als elementar, das ist völlig unstrittig.

    Na wenn du das sagst…
    Ich weiß nur von sehr wenigen Sachen (eigentlich gar keinen), die völlig unstrittig sind.

    an einem Elektron die Zeit ablesen. Wie erkennt man das an ihn?

    Frequenz?

  119. #119 Steffmann
    6. Juni 2017

    @Frantischek:

    Er ist in keinem Semester. Wenn du seine Aussaugen nochmal durchliest, wirst Du feststellen, dass da wenig Substanz war. Mich wundert nur, dass ihr solange gebraucht habt, dieses Mal. Ihr werdet alt und träge, kann das sein ? 😉

  120. #120 Steffmann
    6. Juni 2017

    Spätestens da war Schluss für mich:

    Boltzmann war ein Stochastiker. Das hat mit dem Thema hier nichts zu tun. Auch wenn Du mit dem Hinweis, dass er Entropie erwähnte, Recht hast.

    Er war nicht einfach ein Stochastiker und er hat die Entropie nicht nur mal so erwähnt, sondern ihr eine neue Sichtweise, eine neue Definition gegeben. Er hat den zweiten Hauptsatz der Thermodynamik aus statistischen Betrachtungen abgeleitet.

  121. #121 Physik-Fan
    7. Juni 2017

    @Frantischek
    @Streffmann

    Ihr zwei seid ja lustig. Wir wollen doch schön sachlich bleiben oder? Antwortet im Detail und sachgerecht oder lasst es. Dass Ihr zu Polemik greift, zeugt nicht von Argumentationsstärke, sondern eher vom Gegenteil, dem Fehlen von Argumenten.

    Wer bezweifelt, dass man heute das Elektron als elementar betrachtet, hat arge Wissensmängel über Teilchenphysik. Kennt Ihr einen experimentellen Befund, der auf eine innere Struktur hindeutet? Ich bin schon ganz gespannt …

    Dass man aus der Ruhemasse ein Zeitsignal erhalten kann, ist sensationell. Habe ich noch nie gehört. Irgendwie muss dann das Ticken des Elektrons zu registrieren sein. Wie geht das? Wo hat man das gemacht?

    Dass Boltzmann nur so ein Stochastiker war, der irgendwas herumgerechnet hat, ist auch sehr überraschend. Die Aussage “Wenden wir dies auf den zweiten Hauptsatz an, so können wir diejenige Größe, welche man gewöhnlich als die Entropie zu bezeichnen pflegt, mit der Wahrscheinlichkeit des betreffenden Zustandes identifizieren.” stammt dann bestimmt nicht von ihm. Die Geschichtsschreibung der Physik ist falsch! Und die berühmte Formel über den Zusammenhang zwischen Entropie und Wahrscheinlichkeit hat sicher ein Witzbold oder so an seinem Grab angebracht. Alles klar!

  122. #122 Physik-Fan
    7. Juni 2017

    @Alderamin #111

    Ja, das ist einfach die Vergangenheit. Wenn der Zeitpfeil zu uns führt, ist es unsere Vergangenheit, sonst die eines anderen Orts.Die Blase fängt als Punkt an und dehnt sich dann aus (aus der Außensicht). Irgendwann (Außenzeit) kommt der Moment, wo der für unseren Ort relevante Urknall stattfindet und von da an läuft unser Zeitpfeil nach der Innensicht los und geht ins Innere der Blase hinein. Die Zeit vergeht 13,8 Milliarden Jahre lang und dann sind wir am heutigen Ende des Pfeils. Aus unserer Sicht liegen gleiche Zeiten auf Kugelschalen innerhalb der Blase. Jede solche Schale ist allerdings tatsächlich ein 3D-Raum.

    Das bekräftigt meinen Eindruck einer mehr konstruierten Sache. Ist das alles zwangsläufig, z.B. dass der Zeitpfeil so ist und nicht anders ist? Folgt das streng oder ist es eine (weitere) Hypothese?

    Gemäß Deiner Beschreibung klingt es so, dass die Außengrenze (= Urknall) Orte im Außenraum überschreitet. Aber es ist doch so, dass der Raum expandiert, innerhalb und außerhalb der Blase. Es entsteht einfach neuer Raum, da wird nichts überschritten.

    Generell stelle ich mir betreffend der Plausibilität einer Theorie immer die Fragen:

    – Warum sollte es so sein? Was sind die Argumente, die dafür sprechen? Dass was Brauchbares herauskommt, ist dabei gar nicht so auschlaggebend, denn es könnte auch anders gehen. Die Physikgeschichte ist voll von Beispielen dafür.

    – Gibt es experimentell überprüfbare Vorhersagen? Sind sie verifiziert?

    Kann das Ballonbild denn einen unendlichen Raum zur Zeit des Urknalls abbilden?

    Natürlich nicht im (theoretisch) punktförmigen Anfangszustand, aber danach, selbst wenn die Blase noch winzig klein ist, kann ihr Rand ein geschlossener 3-D-Raum sein, z.B. eine 3-D-Sphäre.

  123. #123 Alderamin
    7. Juni 2017

    @Physik-Fan

    Das bekräftigt meinen Eindruck einer mehr konstruierten Sache. Ist das alles zwangsläufig, z.B. dass der Zeitpfeil so ist und nicht anders ist? Folgt das streng oder ist es eine (weitere) Hypothese?

    Ich hatte Dir doch den Link auf die Aguirre-Seite gegeben. Alles, was ich über das Modell weiß, ist was da steht. Darüber hinaus kann ich Dir nichts erklären, da müsstest Du den Autor direkt fragen.

    Gemäß Deiner Beschreibung klingt es so, dass die Außengrenze (= Urknall) Orte im Außenraum überschreitet. Aber es ist doch so, dass der Raum expandiert, innerhalb und außerhalb der Blase. Es entsteht einfach neuer Raum, da wird nichts überschritten.

    Ich habe m.W.n. nirgendwo behauptet, dass sich die Blase in den Außenraum ausdehnt (und das auch nicht gemeint). Um sich das ganze bildlich vorzustellen, kommt man allerdings nicht umhin, eine wachsende Blase abzubilden. Der Raum drumherum wächst im übrigen auch, rasend viel schneller.

    – Warum sollte es so sein? Was sind die Argumente, die dafür sprechen?
    – Gibt es experimentell überprüfbare Vorhersagen? Sind sie verifiziert?

    Weil der Raum keine nachweisbare Krümmung hat und damit potenziell unendlich groß ist (und es damit immer schon gewesen sein muss). In dem Modell geht es genau um diesen Punkt. Dass der Raum flach ist, haben WMAP und PLANCK immer wieder mit zunehmender Genauigkeit nachgewiesen. Es gibt allerdings auch endliche Mannigfaltigkeiten, die flach sind. Ein Torus ist flach. Theoretisch könnte man nach Bildern der Milchstraße oder ihrer Umgebung in der Ferne suchen, die eine Runde um den Umfang geschafft haben. Oder nach Mustern in der Hintergrundstrahlung.

    Natürlich nicht im (theoretisch) punktförmigen Anfangszustand, aber danach, selbst wenn die Blase noch winzig klein ist, kann ihr Rand ein geschlossener 3-D-Raum sein, z.B. eine 3-D-Sphäre.

    Das war nicht die Frage. Eine geschlossene 4D-Sphäre mit 3D-Hülle hat ein endliches Volumen und eine Krümmung, die potenziell messbar ist. Dieser Raum wäre jedenfalls nicht flach.

  124. #124 Frantischek
    7. Juni 2017

    Wie geht das? Wo hat man das gemacht?

    Wenn du bis jetzt nicht verstanden hast was eine Frequenz ist, und was sie mit der Ruhemasse zu tun hat, dann soll es dir mal schön wer erklären der dafür bezahlt wird.

  125. #125 Physik-Fan
    8. Juni 2017

    @Frantischek #124

    Wenn du bis jetzt nicht verstanden hast was eine Frequenz ist, und was sie mit der Ruhemasse zu tun hat, dann soll es dir mal schön wer erklären der dafür bezahlt wird.

    Um es klarer zu machen, habe ich konkrete Fragen gestellt. Denke mal darüber nach. Vielleicht merkst Du dann, welchen Quatsch Du sagst.

    Um eine Frequenz von irgendwas registrieren zu können, muss ja vom Elektron eine Strahlung ausgehen. Ein freies Elektron in gleichförmiger Bewegung gibt keine Strahlung ab. Das tut es z.B. wenn es beschleunigt wird und wenn es im Atom “zurückspringt”. Das liegt an der elektromagnet. WW. An der nimmt das Elektron wegen seiner elektr. Ladung teil. Seine Ruhemasse hat damit nichts zu tun. Man könnte allgemein statt Elektron “geladenes Teilchen” sagen.

    Dass das Elektron als elementar zu betrachten ist, sagt das Standardmodell der Teilchenphysik. Die Kenntnis zumindest der Grundzüge davon, ist unabdingbar. Es beschreibt u.a. die fundamentalen Materiebausteine (nach heutiger Kenntnis). Schon mal was von Quarks und Leptonen gehört? Und wozu gehört das Elektron?

  126. #126 Physik-Fan
    8. Juni 2017

    @Alderamin #123

    Das Argument der Flachheit ist gut, akzeptiert. Aber Du sagst ja selbst, dass die Geometrie im Rand eines geschlossenen Körpers auch euklidisch sein kann. Bei der 3-Sphäre ist das natürlich nicht so, aber sie kann lokal, in einer genügend kleinen Umgebung, mit guter Näherung euklidisch sein. Dazu muss Universum nur groß genug sein. Allerdings muss das schon nach 380.000 Jahren so gewesen, so weit reicht ja unser Sichthorizont. Ob die klassische Expansion (nicht-inflationär) das hergibt, das bezweifele ich eher. Erst inflationär kommt es ja zu einem viel größeren Universum. Also die 3-Sphäre ist wohl kein so gutes Modell.

  127. #127 Physik-Fan
    8. Juni 2017

    @Physik-Fan #126

    Also die 3-Sphäre ist wohl kein so gutes Modell.

    Ergänzung: Das gilt für die Blasen, weil sie durch lokalen Zerfall des Falschen Vakuums entstehen, anfänglich klein sind und ihre Ausdehnung nur mit normalem Vakuum erfolgt. Ohne Multiversum, wenn das Falsche Vakuum komplett zu einem einzigen Universum zerfällt, kann das Universum schon groß genug sein. Ich weiß aber nicht, ob dieser Zerfall überhaupt noch ein haltbares Szenario ist.

  128. #128 Alderamin
    9. Juni 2017

    @Physik-Fan

    Das Argument der Flachheit ist gut, akzeptiert. Aber Du sagst ja selbst, dass die Geometrie im Rand eines geschlossenen Körpers auch euklidisch sein kann.

    Die Frage ist, warum das Weltall so eine merkwürdige Topologie haben soll. Unendlich groß und flach ist die einfachste Annahme.

    Ich weiß aber nicht, ob dieser Zerfall überhaupt noch ein haltbares Szenario ist.

    Laut Wikipedia zur Inflation ist das durchaus noch aktuell.

  129. #129 branworld
    Luxemburg
    20. Oktober 2017

    Ambivalent schreibt in # 29 :
    ” Es gibt verschiedene Arten, wie sich zusätzliche Dimensionen zeigen können. Soweit ich weiß, ist zur Zeit die Hypothese der “aufgerollten Dimensionen” die führende. Demnach gäbe es die Raum- und Zeitdimensionen, die wir makroskopisch erfassen können, aber zusätzlich gäbe es noch Dimensionen von beschränkter Ausdehnung, die sich aber eher nur im extremen Mikrokosmos zeigen. Das würde nicht verschiedene Universen bedeuten, sondern eher ein 4dimensionales Universum mit einer 10dimensionalen Feinstruktur. ”

    Frage :
    Könnte man daraus ableiten, dass : Gefaltete Dimensionen unter Umständen eine Masse haben – etwa die Masse eines Baryons. Bei der Annahme dass diese clusterartige Strukturen bilden – ohne überhaupt untereinander interagieren zu müssen – ein Gravitationspotential bilden, welches den Anforderungen an die ” dunkle Materie ” ziemlich nahe käme… ?